Site Loader

Содержание

ЭДС. Закон Ома для полной цепи

Автор статьи — профессиональный репетитор, автор учебных пособий для подготовки к ЕГЭ Игорь Вячеславович Яковлев

Темы кодификатора ЕГЭ: электродвижущая сила, внутреннее сопротивление источника тока, закон Ома для полной электрической цепи.

До сих пор при изучении электрического тока мы рассматривали направленное движение свободных зарядов во внешней цепи, то есть в проводниках, подсоединённых к клеммам источника тока.

Как мы знаем, положительный заряд :

• уходит во внешнюю цепь с положительной клеммы источника;

• перемещается во внешней цепи под действием стационарного электрического поля, создаваемого другими движущимися зарядами;

• приходит на отрицательную клемму источника, завершая свой путь во внешней цепи.

Теперь нашему положительному заряду нужно замкнуть свою траекторию и вернуться на положительную клемму. Для этого ему требуется преодолеть заключительный отрезок пути — внутри источника тока от отрицательной клеммы к положительной. Но вдумайтесь: идти туда ему совсем не хочется! Отрицательная клемма притягивает его к себе, положительная клемма его от себя отталкивает, и в результате на наш заряд внутри источника действует электрическая сила , направленная

против движения заряда (т.е. против направления тока).

Сторонняя сила

Тем не менее, ток по цепи идёт; стало быть, имеется сила, «протаскивающая» заряд сквозь источник вопреки противодействию электрического поля клемм (рис. 1).

Рис. 1. Сторонняя сила

Эта сила называется сторонней силой; именно благодаря ей и функционирует источник тока. Сторонняя сила не имеет отношения к стационарному электрическому полю — у неё, как говорят,

неэлектрическое происхождение; в батарейках, например, она возникает благодаря протеканию соответствующих химических реакций.

Обозначим через работу сторонней силы по перемещению положительного заряда q внутри источника тока от отрицательной клеммы к положительной. Эта работа положительна, так как направление сторонней силы совпадает с направлением перемещения заряда. Работа сторонней силы называется также работой источника тока.

Во внешней цепи сторонняя сила отсутствует, так что работа сторонней силы по перемещению заряда во внешней цепи равна нулю. Поэтому работа сторонней силы по перемещению заряда вокруг всей цепи сводится к работе по перемещению этого заряда только лишь внутри источника тока. Таким образом, — это также работа сторонней силы по перемещению заряда

по всей цепи.

Мы видим, что сторонняя сила является непотенциальной — её работа при перемещении заряда по замкнутому пути не равна нулю. Именно эта непотенциальность и обеспечивает циркулирование электрического тока; потенциальное электрическое поле, как мы уже говорили ранее, не может поддерживать постоянный ток.

Опыт показывает, что работа прямо пропорциональна перемещаемому заряду . Поэтому отношение уже не зависит от заряда и является количественной характеристикой источника тока. Это отношение обозначается :

(1)

Данная величина называется электродвижущей силой (ЭДС) источника тока. Как видим, ЭДС измеряется в вольтах (В), поэтому название «электродвижущая сила» является крайне неудачным. Но оно давно укоренилось, так что приходится смириться.

Когда вы видите надпись на батарейке: «1,5 В», то знайте, что это именно ЭДС. Равна ли эта величина напряжению, которое создаёт батарейка во внешней цепи? Оказывается, нет! Сейчас мы поймём, почему.

Закон Ома для полной цепи

Любой источник тока обладает своим сопротивлением , которое называется внутренним сопротивлением этого источника. Таким образом, источник тока имеет две важных характеристики: ЭДС и внутреннее сопротивление.

Пусть источник тока с ЭДС, равной , и внутренним сопротивлением подключён к резистору (который в данном случае называется внешним резистором, или внешней нагрузкой, или полезной нагрузкой). Всё это вместе называется полной цепью (рис. 2).

Рис. 2. Полная цепь

Наша задача — найти силу тока в цепи и напряжение на резисторе .

За время по цепи проходит заряд . Согласно формуле (1) источник тока совершает при этом работу:

(2)

Так как сила тока постоянна, работа источника целиком превращается в теплоту, которая выделяется на сопротивлениях и . Данное количество теплоты определяется законом Джоуля–Ленца:

(3)

Итак, , и мы приравниваем правые части формул (2) и (3):

После сокращения на получаем:

Вот мы и нашли ток в цепи:

(4)

Формула (4) называется законом Ома для полной цепи.

Если соединить клеммы источника проводом пренебрежимо малого сопротивления , то получится

короткое замыкание. Через источник при этом потечёт максимальный ток — ток короткого замыкания:

Из-за малости внутреннего сопротивления ток короткого замыкания может быть весьма большим. Например, пальчиковая батарейка разогревается при этом так, что обжигает руки.

Зная силу тока (формула (4)), мы можем найти напряжение на резисторе с помощью закона Ома для участка цепи:

(5)

Это напряжение является разностью потенциалов между точками и (рис. 2). Потенциал точки равен потенциалу положительной клеммы источника; потенциал точки равен потенциалу отрицательной клеммы. Поэтому напряжение (5) называется также

напряжением на клеммах источника.

Мы видим из формулы (5), что в реальной цепи будет — ведь умножается на дробь, меньшую единицы. Но есть два случая, когда .

1. Идеальный источник тока. Так называется источник с нулевым внутренним сопротивлением. При формула (5) даёт .

2. Разомкнутая цепь. Рассмотрим источник тока сам по себе, вне электрической цепи. В этом случае можно считать, что внешнее сопротивление бесконечно велико: . Тогда величина неотличима от , и формула (5) снова даёт нам .

Смысл этого результата прост: если источник не подключён к цепи, то вольтметр, подсоединённый к полюсам источника, покажет его ЭДС.

КПД электрической цепи

Нетрудно понять, почему резистор называется полезной нагрузкой. Представьте себе, что это лампочка. Теплота, выделяющаяся на лампочке, является полезной, так как благодаря этой теплоте лампочка выполняет своё предназначение — даёт свет.

Количество теплоты, выделяющееся на полезной нагрузке за время , обозначим .

Если сила тока в цепи равна , то

Некоторое количество теплоты выделяется также на источнике тока:

Полное количество теплоты, которое выделяется в цепи, равно:

КПД электрической цепи — это отношение полезного тепла к полному:

КПД цепи равен единице лишь в том случае, если источник тока идеальный .

Закон Ома для неоднородного участка

Простой закон Ома справедлив для так называемого однородного участка цепи — то есть участка, на котором нет источников тока. Сейчас мы получим более общие соотношения, из которых следует как закон Ома для однородного участка, так и полученный выше закон Ома для полной цепи.

Участок цепи называется неоднородным, если на нём имеется источник тока. Иными словами, неоднородный участок — это участок с ЭДС.

На рис. 3показан неоднородный участок, содержащий резистор и источник тока. ЭДС источника равна , его внутреннее сопротивление считаем равным нулю (усли внутреннее сопротивление источника равно , можно просто заменить резистор на резистор ).

Рис. 3. ЭДС «помогает» току:

Сила тока на участке равна , ток течёт от точки к точке . Этот ток не обязательно вызван одним лишь источником . Рассматриваемый участок, как правило, входит в состав некоторой цепи (не изображённой на рисунке), а в этой цепи могут присутствовать и другие источники тока. Поэтому ток является результатом совокупного действия всех источников, имеющихся в цепи.

Пусть потенциалы точек и равны соответственно и .

Подчеркнём ещё раз, что речь идёт о потенциале стационарного электрического поля, порождённого действием всех источников цепи — не только источника, принадлежащего данному участку, но и, возможно, имеющихся вне этого участка.

Напряжение на нашем участке равно: . За время через участок проходит заряд , при этом стационарное электрическое поле совершает работу:

Кроме того, положительную работу совершает источник тока (ведь заряд прошёл сквозь него!):

Сила тока постоянна, поэтому суммарная работа по продвижению заряда , совершаемая на участке стационарным элетрическим полем и сторонними силами источника, целиком превращается в тепло: .

Подставляем сюда выражения для , и закон Джоуля–Ленца:

Сокращая на , получаем закон Ома для неоднородного участка цепи:

(6)

или, что то же самое:

(7)

Обратите внимание: перед стоит знак «плюс». Причину этого мы уже указывали — источник тока в данном случае совершает положительную работу, «протаскивая» внутри себя заряд от отрицательной клеммы к положительной. Попросту говоря, источник «помогает» току протекать от точки к точке .

Отметим два следствия выведенных формул (6) и (7).

1. Если участок однородный, то . Тогда из формулы (6) получаем — закон Ома для однородного участка цепи.

2. Предположим, что источник тока обладает внутренним сопротивлением . Это, как мы уже упоминали, равносильно замене на :

Теперь замкнём наш участок, соединив точки и . Получим рассмотренную выше полную цепь. При этом окажется, что и предыдущая формула превратится в закон Ома для полной цепи:

Таким образом, закон Ома для однородного участка и закон Ома для полной цепи оба вытекают из закона Ома для неоднородного участка.

Может быть и другой случай подключения, когда источник «мешает» току идти по участку. Такая ситуация изображена на рис. 4. Здесь ток, идущий от к , направлен против действия сторонних сил источника.

Рис. 4. ЭДС «мешает» току:

Как такое возможно? Очень просто: другие источники, имеющиеся в цепи вне рассматриваемого участка, «пересиливают» источник на участке и вынуждают ток течь против . Именно так происходит, когда вы ставите телефон на зарядку: подключённый к розетке адаптер вызывает движение зарядов против действия сторонних сил аккумулятора телефона, и аккумулятор тем самым заряжается!

Что изменится теперь в выводе наших формул? Только одно — работа сторонних сил станет отрицательной:

Тогда закон Ома для неоднородного участка примет вид:

(8)

или:

где по-прежнему — напряжение на участке.

Давайте соберём вместе формулы (7) и (8) и запишем закон Ома для участка с ЭДС следующим образом:

Ток при этом течёт от точки к точке . Если направление тока совпадает с направлением сторонних сил, то перед ставится «плюс»; если же эти направления противоположны, то ставится «минус».

формула через силу тока, индуктивность или площадь, единица измерения в физике

Что такое ЭДС индукции — когда возникает, при каких условиях

Определение

Электродвижущая сила, ЭДС — физическая величина, описывающая работу любых сил, которые действуют в квазистационарных цепях постоянного или переменного тока, за исключением диссипативных и электростатических сил.

При замкнутой цепи можно найти ЭДС, воспользовавшись законом Ома:

\(\varepsilon\;=\;I\;\times\;(R\;+\;r).\)

R здесь — сопротивление цепи, r — внутреннее сопротивление источника.
Создание Алессандро Вольтой надежного источника электричества, гальванического элемента, и открытие Хансом Кристианом Эрстедом магнитного действия электрического тока послужили толчком к интенсивному развитию техники электрических измерений в XIX веке.

Осторожно! Если преподаватель обнаружит плагиат в работе, не избежать крупных проблем (вплоть до отчисления). Если нет возможности написать самому, закажите тут.

Выдающаяся роль здесь принадлежит немецкому физику Георгу Симону Ому. Для определения силы тока он использовал принцип крутильных весов Кулона. На длинной тонкой нити подвешено горизонтальное коромысло с заряженным шариком на конце. Второй заряд закреплен на спице, пропущенной сквозь крышку весов.

При их взаимодействии коромысло поворачивается. Вращение головки в верхней части весов закручивало нить, возвращая коромысло в исходное состояние. По углу закручивания можно рассчитать силу взаимодействия зарядов в зависимости от расстояния между ними.

Ом по величине угла закрутки судил о силе тока I в проводнике, т. е. количестве электричества, перенесенном через поперечное сечение проводника за единицу времени.

В качестве основной характеристики источника тока Ом брал величину напряжения \varepsilon на электродах гальванического элемента при разомкнутой цепи. Эту величину \varepsilon он назвал электродвижущей силой, сокращенно ЭДС.

Движущиеся заряды создают вокруг себя магнитное поле. Однако действующая в нем на магнит или другой ток сила отличается от электрической своим направлением — магнитная стрелка старается развернуться перпендикулярно проводу.

Изучение действующей на другой ток силы переросло в отдельное исследование с неожиданным результатом: сила оказалась направленной всегда перпендикулярно внесенному в магнитное поле проводнику, который для простоты исследования был прямолинейным.

Математическое выражение для этой силы, названной силой Ампера, проще всего записать в виде векторного произведения:

\(d\overrightarrow F\;=\;Id\overrightarrow l\;\times\;\overrightarrow B\).

I здесь — сила тока, протекающего через проводник; l — вектор длины проводника, направленный в ту же сторону, куда течет ток; В — характеристика поля. Величина В называется магнитной индукцией и является аналогом электрической напряженности.

Максвелл поставил целью создать теорию эфира, связав его механические характеристики с электрическими и магнитными силами. Тщательно изучив труды Фарадея, он пришел к выводу, что напряженность \(\overrightarrow Е\) электрического поля объясняется упругими напряжениями в эфире, а магнитная индукция \(\overrightarrow B\) — его вихревыми движениями.

Рассматривая замкнутый проводящий контур С, где действует ЭДС индукции \(\varepsilon_i\), Максвелл для получения числа силовых линий магнитного потока \(\triangle Ф\), пересекаемых контуром за время \triangle t, «натягивал» на него некую поверхность S, разбитую на элементарные площадки \(\triangle S\), и отождествлял Ф с магнитным потоком сквозь всю поверхность. Математически это можно выразить так:

\(Ф\;=\;\sum_{\triangle S}\;\;B\triangle S. \)

Объединив это соотношение с идеей Фарадея, Максвелл пришел к собственной формуле:

\(\varepsilon_i\;=\;-\;\frac1с\;\times\;\frac{dФ}{dt}. \)

Выбор коэффициента пропорциональности \(\alpha\) здесь обусловлен необходимостью согласования формулы с законом Био — Савара — Лапласа, в котором появляется та же электродинамическая постоянная с.

Определение

Электродинамическая постоянная с — универсальная постоянная, равная скорости распространения электромагнитных волн в вакууме.

Но в опытах Фарадея ЭДС индукции регистрировалась как в движущемся, так и в покоящемся проводящем контуре С, если последний находился в переменном магнитном поле. И здесь встал вопрос, что конкретно перемещает заряды в неподвижном проводнике.

Само по себе магнитное поле не воздействует на заряды, находящиеся в покое, из чего следует: условие возникновения индукционного тока — возникающее в контуре электрическое поле \overrightarrow Е. Так как электростатическое поле в замкнутом контуре не совершает работы, значит, происходит работа вихревого поля, и она равна ЭДС индукции:

\(\varepsilon_i\;=\;\underset С{\oint\;}\;(\overrightarrow{Е\;}\times\;d\overrightarrow l)\)

Определение

Самоиндукция — частный случай магнитной индукции, возникновение ЭДС индукции в проводящем контуре, когда в нем меняется ток.

Источником энергии, возникающей в цепи, является в этом случае запас энергии магнитного поля. Полное количество выделившейся джоулевой теплоты можно вычислить, изобразив на графике зависимость магнитного потока Ф(I) от силы тока I:

Источник: physics.ru

ЭДС в быту, как обозначается, единицы измерения

В быту явление электромагнитной индукции используют для изменения величины напряжения тока в трансформаторах и дросселях. На принципе магнитной индукции работают электрические счетчики, реле мощности, успокоительные системы стрелочных измерительных приборов.

Существуют также магнитные газовые генераторы, в которых благодаря магнитному полю возникает электродвижущая сила, создающая ток.

Электродвижущая сила индукции в системе СИ измеряется в вольтах. Просто электродвижущая сила обозначается греческой буквой \(\varepsilon \), электродвижущая сила индукции —\( \varepsilon_i.\)

Законы Фарадея и Ленца

Фарадей опытным путем выяснил, что при пересечении проводником магнитных силовых линий по нему проходит заряд \(\triangle Q\).  Он связан с числом пересеченных силовых линий \( \triangle Ф\) и электрическим сопротивлением контура R, что выражается законом Фарадея:

\(\triangle Q\;=\;\alpha\frac{\triangle Ф}R. \)

Соприкосновение поля и проводника вызвано либо движением проводника, либо изменениями самого магнитного поля. 

Саму электродвижущую силу индукции, связанную с сопротивлением контура и силой тока согласно закону Ома, можно найти по формуле

\(\varepsilon_i\;=\;\alpha\frac{\triangle Ф}{\triangle t}. \)

\(\triangle t\) здесь — время, за которое проходит через поперечное сечение проводника количество электричества \(\triangle Q.\)
Ленц доказал, что индукционный ток всегда направлен так, чтобы противодействовать вызвавшей его причине. Согласно правилу Ленца, в вышеприведенном соотношении следует выбрать отрицательный знак, считая коэффициент\( \alpha \) положительным: 

\(\varepsilon_i\;=\;-\;\alpha\frac{\triangle Ф}{\triangle t}. \)

Как рассчитать электродвижущую силу индукции, формулы

Через магнитный поток

\(\varepsilon_i\;=\;-\;\alpha\frac{\triangle Ф}{\triangle t}. \)

Через силу тока

ЭДС самоиндукции зависит от изменения силы тока, при этом магнитный поток собственного поля через цепь пропорционален току в ней:

\(\varepsilon_{is\;}\;=\;-\;L\frac{\triangle I}{\triangle t}. \)

L здесь — индуктивность проводника.

Через сопротивление

Для ЭДС индукции уравнение закона Ома можно переписать в виде:
\(\varepsilon_{i\;}\;=\;IR\;-\;\varepsilon.\)

Через угловую скорость

\(\varepsilon_i\;=\;В\omega SN\sin\left(\alpha\right). \)

B здесь — индукция магнитного поля, \(\omega\) — угловая скорость вращения рамки, S — площадь рамки, N — число витков, \(\alpha\) — угол между векторами индукции магнитного поля и скорости движения проводника.

Через площадь

Если магнитный поток изменяется без деформации витков, т. е. их количество и площадь не меняются, то можно найти электродвижущую силу индукции через площадь.
Угол \alpha между вектором магнитного поля и нормалью к плоскости витков будет равен:

\(2\mathrm\pi\;\times\;\mathrm v\;\times\;\mathrm t. \)Полный магнитный поток в момент времени t будет равен:

\(\psi_B\;=\;N\;\times\;B\;\times\;S\;\times\;\cos\left(\alpha\right)=\;N\;\times\;B\;\times\;S\;\times\;\cos\left(2\mathrm\pi\;\times\;\mathrm v\;\times\;\mathrm t\right).\)

Тогда \(\varepsilon_i\;=\;-\;\frac{d\psi_B}{dt}=\;2\mathrm{pivNBSsin}\left(2\mathrm{pivt}\right).\)

Урок 31. закон ома для полной цепи — Физика — 10 класс

Физика, 10 класс

Урок 31. Закон Ома для полной цепи

Перечень вопросов, рассматриваемых на уроке:

1) закон Ома для полной цепи;

2) связь ЭДС с внутренним сопротивлением;

3) короткое замыкание;

4) различие между ЭДС, напряжением и разностью потенциалов.

Глоссарий по теме

Электрическая цепь – набор устройств, которые соединены проводниками, предназначенный для протекания тока.

Электродвижущая сила – это отношение работы сторонних сил при перемещении заряда по замкнутому контуру к абсолютной величине этого заряда.

Закон Ома для полной цепи: сила тока в полной цепи равна отношению ЭДС цепи к ее полному сопротивлению:

Основная и дополнительная литература по теме урока:

1. Мякишев Г. Я., Буховцев Б. Б., Сотский Н.Н. Физика. 10 класс. Учебник для общеобразовательных организаций М.: Просвещение, 2017. С. 348 – 354.

2.Рымкевич А. П. Сборник задач по физике. 10-11 класс. — М.: Дрофа, 2009. С. 106-108.

Теоретический материал для самостоятельного изучения

Любые силы, которые действуют на электрически заряженные частицы, кроме сил электростатического происхождения (т.е. кулоновских), называют сторонними силами. Сторонние силы приводят в движение заряженные частицы внутри всех источников тока.

Действие сторонних сил характеризуется важной физической величиной электродвижущей силой (ЭДС). Электродвижущая сила в замкнутом контуре — отношение работы сторонних сил при перемещении заряда вдоль контура к заряду.

В источнике тока из-за действием сторонних сил происходит разделение зарядов. Так как они движутся, они взаимодействуют с ионами кристаллов и электролитов и отдают им часть своей энергии. Это приводит к уменьшению силы тока, таким образом, источник тока обладает сопротивлением, которое называют внутренним r.

Закон Ома для замкнутой цепи связывает силу тока в цепи, ЭДС и полное сопротивление цепи:

Сила тока в полной цепи равна отношению ЭДС цепи к ее полному сопротивлению

Короткое замыкание

При коротком замыкании, когда внешнее сопротивление стремится к нулю , сила тока в цепи определяется именно внутренним сопротивлением и может оказаться очень большой . И тогда провода могут расплавиться, что может привести к опасным последствиям.

Примеры и разбор решения заданий:

1. К каждой позиции первого столбца подберите соответствующую позицию второго:

ФИЗИЧЕСКИЕ ВЕЛИЧИНЫ

ФОРМУЛЫ

Электродвижущая сила

Сила тока

Сопротивление

Разность потенциалов

Решение.

Электродвижущая сила гальванического элемента есть величина, численно равная работе сторонних сил при перемещении единичного положительного заряда внутри элемента от одного полюса к другому.

Работа сторонних сил не может быть выражена через разность потенциалов, так как сторонние силы непотенциальны и их работа зависит от формы траектории перемещения зарядов.

ЭДС определяется по формуле:

Сила тока определяется по формуле:

Сопротивление определяется по формуле:

Разность потенциалов определяется по формуле:

Правильный ответ:

ФИЗИЧЕСКИЕ ВЕЛИЧИНЫ

ФОРМУЛЫ

Электродвижущая сила

Сила тока

Сопротивление

Разность потенциалов

2. ЭДС батарейки карманного фонарика — 3,7 В, внутреннее сопротивление 1,5 Ом. Батарейка замкнута на сопротивление 11,7 Ом. Каково напряжение на зажимах батарейки?

Решение:

Напряжение рассчитывается по формуле:

Чтобы найти силу тока применим закон Ома для полной цепи:

Делаем расчёт:

Ответ: U = 3,28 В.

Определить силу тока в проводнике R1, если ЭДС источника 14 В, его внутреннее сопротивление

Условие задачи:

Определить силу тока в проводнике \(R_1\), если ЭДС источника 14 В, его внутреннее сопротивление 1 Ом, \(R_1=R_3=10\) Ом, \(R_2=5\) Ом.

Задача №7.2.13 из «Сборника задач для подготовки к вступительным экзаменам по физике УГНТУ»

Дано:

\(\rm E=14\) В, \(r=1\) Ом, \(R_1=R_3=10\) Ом, \(R_2=5\) Ом, \(I_1-?\)

Решение задачи:

В узле A ток \(I\) разделяется на два тока: ток \(I_1\) и ток \(I_2\), поэтому верно записать следующее (строго говоря, это первый закон Кирхгофа):

\[I = {I_1} + {I_2}\;\;\;\;(1)\]

Так как два сопротивления \(R_1\) и \(R_2\) соединены параллельно с сопротивлением \(R_3\) (то есть напряжения на них одинаковы), то справедливо равенство:

\[{I_1}\left( {{R_1} + {R_2}} \right) = {I_2}{R_3}\]

Выразим из этого равенства ток \(I_2\):

\[{I_2} = {I_1}\frac{{{R_1} + {R_2}}}{{{R_3}}}\]

Полученное подставим в (1), тогда:

\[I = {I_1} + {I_1}\frac{{{R_1} + {R_2}}}{{{R_3}}}\]

\[I = {I_1}\frac{{{R_1} + {R_2} + {R_3}}}{{{R_3}}}\]

Отсюда выразим искомый ток \(I_1\):

\[{I_1} = \frac{{I{R_3}}}{{{R_1} + {R_2} + {R_3}}}\;\;\;\;(2)\]

Чтобы узнать общий ток \(I\), запишем закон Ома для полной цепи:

\[I = \frac{{\rm E}}{{R + r}}\;\;\;\;(3)\]

Здесь \(R\) – внешнее сопротивление цепи, которое можно найти по формуле (напомним, что эквивалентное сопротивление двух последовательно соединенных резисторов равно сумме этих сопротивлений):

\[\frac{1}{R} = \frac{1}{{{R_1} + {R_2}}} + \frac{1}{{{R_3}}}\]

\[\frac{1}{R} = \frac{{{R_1} + {R_2} + {R_3}}}{{\left( {{R_1} + {R_2}} \right){R_3}}}\]

\[R = \frac{{\left( {{R_1} + {R_2}} \right){R_3}}}{{{R_1} + {R_2} + {R_3}}}\;\;\;\;(4)\]

В итоге, сначала найдем сопротивление внешней цепи \(R\) по формуле (4), далее найдем ток \(I\) в цепи по формуле (3), а потом уже посчитаем искомый ток во втором сопротивлении \(I_2\) по формуле (2).

\[R = \frac{{\left( {10 + 5} \right) \cdot 10}}{{10 + 5 + 10}} = 6\;Ом\]

\[I = \frac{{14}}{{6 + 1}} = 2\;А\]

\[{I_1} = \frac{{2 \cdot 10}}{{10 + 5 + 10}} = 0,8\;А = 800\;мА\]

Ответ: 800 мА.

Если Вы не поняли решение и у Вас есть какой-то вопрос или Вы нашли ошибку, то смело оставляйте ниже комментарий.

Глава 23. Закон электромагнитной индукции

Если в магнитном поле находится замкнутый проводящий контур, не содержащий источников тока, то при изменении магнитного поля в контуре возникает электрический ток. Это явление называется электромагнитной индукцией. Появление тока свидетельствует о возникновении в контуре электрического поля, которое может обеспечить замкнутое движение электрических зарядов или, другими словами, о возникновении ЭДС. Электрическое поле, которое возникает при изменении поля магнитного и работа которого при перемещении зарядов по замкнутому контуру не равна нулю, имеет замкнутые силовые линии и называется вихревым.

Для количественного описания электромагнитной индукции вводится понятие магнитного потока (или потока вектора магнитной индукции) через замкнутый контур. Для плоского контура, расположенного в однородном магнитном поле (а только такие ситуации и могут встретиться школьникам на едином государственном экзамене), магнитный поток определяется как

(23.1)

где — индукция поля, — площадь контура, — угол между вектором индукции и нормалью (перпендикуляром) к плоскости контура (см. рисунок; перпендикуляр к плоскости контура показан пунктиром). Единицей магнитного потока в международной системе единиц измерений СИ является Вебер (Вб), который определяется как магнитный поток через контур площади 1 м2 однородного магнитного поля с индукцией 1 Тл, перпендикулярной плоскости контура.

Величина ЭДС индукции , возникающая в контуре при изменении магнитного потока через этот контур, равна скорости изменения магнитного потока

(23.2)

Здесь — изменение магнитного потока через контур за малый интервал времени . Важным свойством закона электромагнитной индукции (23.2) является его универсальность по отношению к причинам изменения магнитного потока: магнитный поток через контур может меняться из-за изменения индукции магнитного поля, изменения площади контура или изменения угла между вектором индукции и нормалью, что происходит при вращении контура в поле. Во всех этих случаях по закону (23.2) в контуре будет возникать ЭДС индукции и индукционный ток.

Знак минус в формуле (23.2) «отвечает» за направление тока, возникающего в результате электромагнитной индукции (правило Ленца). Однако понять на языке закона (23.2), к какому направлению индукционного тока приведет этот знак при том или ином изменении магнитного потока через контур, не так-то просто. Но достаточно легко запомнить результат: индукционный ток будет направлен таким образом, что созданное им магнитное поле будет «стремиться» компенсировать то изменение внешнего магнитного поля, которое этот ток и породило. Например, при увеличении потока внешнего магнитного поля через контур в нем возникнет индукционный ток, магнитное поле которого будет направлено противоположно внешнему магнитному полю так, чтобы уменьшить внешнее поле и сохранить, таким образом, первоначальную величину магнитного поля. При уменьшении потока поля через контур поле индукционного тока будет направлено так же, как и внешнее магнитное поле.

Если в контуре с током ток в силу каких-то причин изменяется, то изменяется и магнитный поток через контур того магнитного поля, которое создано самим этим током. Тогда по закону (23.2) в контуре должна возникать ЭДС индукции. Явление возникновения ЭДС индукции в некоторой электрической цепи в результате изменения тока в самой этой цепи называется самоиндукцией. Для нахождения ЭДС самоиндукции в некоторой электрической цепи необходимо вычислить поток магнитного поля, создаваемого этой цепью через нее саму. Такое вычисление представляет собой сложную проблему из-за неоднородности магнитного поля. Однако одно свойство этого потока является очевидным. Поскольку магнитное поле, создаваемого током в цепи, пропорционально величине тока, то и магнитный поток собственного поля через цепь пропорционален току в этой цепи

(23.3)

где — сила тока в цепи, — коэффициент пропорциональности, который характеризует «геометрию» цепи, но не зависит от тока в ней и называется индуктивностью этой цепи. Единицей индуктивности в международной системе единиц СИ является Генри (Гн). 1 Гн определяется как индуктивность такого контура, поток индукции собственного магнитного поля через который равен 1 Вб при силе тока в нем 1 А. С учетом определения индуктивности (23.3) из закона электромагнитной индукции (23.2) получаем для ЭДС самоиндукции

(23.4)

Благодаря явлению самоиндукции ток в любой электрической цепи обладает определенной «инерционностью» и, следовательно, энергией. Действительно, для создания тока в контуре необходимо совершить работу по преодолению ЭДС самоиндукции. Энергия контура с током и равна этой работе. Необходимо запомнить формулу для энергии контура с током

(23.5)

где — индуктивность контура, — сила тока в нем.

Явление электромагнитной индукции широко применяется в технике. На нем основано создание электрического тока в электрических генераторах и электростанциях. Благодаря закону электромагнитной индукции происходит преобразование механических колебаний в электрические в микрофонах. На основе закона электромагнитной индукции работает, в частности, электрическая цепь, которая называется колебательным контуром (см. следующую главу), и которая является основой любой радиопередающей или радиопринимающей техники.

Рассмотрим теперь задачи.

Из перечисленных в задаче 23.1.1 явлений только одно есть следствие закона электромагнитной индукции — появление тока в кольце при проведении сквозь него постоянного магнита (ответ 3). Все остальное — результат магнитного взаимодействия токов.

Как указывалось во введении к настоящей главе, явление электромагнитной индукции лежит в основе работы генератора переменного тока (задача 23.1.2), т.е. прибора, создающего переменный ток, заданной частоты (ответ 2).

Индукция магнитного поля, создаваемого постоянным магнитом, уменьшается с увеличением расстояния до него. Поэтому при приближении магнита к кольцу (задача 23.1.3) поток индукции магнитного поля магнита через кольцо изменяется, и в кольце возникает индукционный ток. Очевидно, это будет происходить при приближении магнита к кольцу и северным, и южным полюсом. А вот направление индукционного тока в этих случаях будет различным. Это связано с тем, что при приближении магнита к кольцу разными полюсами, поле в плоскости кольца в одном случае будет направлено противоположно полю в другом. Поэтому для компенсации этих изменений внешнего поля магнитное поле индукционного тока должно быть в этих случаях направлено по-разному. Поэтому и направления индукционных токов в кольце будут противоположными (ответ 4).

Для возникновения ЭДС индукции в кольце необходимо, чтобы менялся магнитный поток через кольцо. А поскольку магнитная индукция поля магнита зависит от расстояния до него, то в рассматриваемом в задаче 23.1.4 случае поток через кольцо будет меняться, в кольце возникнет индукционный ток (ответ 1).

При вращении рамки 1 (задача 23.1.5) угол между линиями магнитной индукции (а, значит, и вектором индукции) и плоскостью рамки в любой момент времени равен нулю. Следовательно, магнитный поток через рамку 1 не изменяется (см. формулу (23.1)), и индукционный ток в ней не возникает. В рамке 2 индукционный ток возникнет: в положении показанном на рисунке, магнитный поток через нее равен нулю, когда рамка повернется на четверть оборота — будет равен , где — индукция, — площадь рамки. Еще через четверть оборота поток снова будет равен нулю и т.д. Поэтому поток магнитной индукции через рамку 2 изменяется в процессе ее вращения, следовательно, в ней возникает индукционный ток (ответ 2).

В задаче 23.1.6 индукционный ток возникает только в случае 2 (ответ 2). Действительно, в случае 1 рамка при движении остается на одном и том же расстоянии от проводника, и, следовательно, магнитное поле, созданное этим проводником в плоскости рамки, не изменяется. При удалении рамки от проводника магнитная индукция поля проводника в области рамки изменяется, меняется магнитный поток через рамку, и возникает индукционный ток

В законе электромагнитной индукции утверждается, что индукционный ток в кольце будет течь в такие моменты времени, когда изменяется магнитный поток через это кольцо. Поэтому пока магнит покоится около кольца (задача 23.1.7) индукционный ток в кольце течь не будет. Поэтому правильный ответ в этой задаче — 2.

Согласно закону электромагнитной индукции (23.2) ЭДС индукции в рамке определяется скоростью изменения магнитного потока через нее. А поскольку по условию задачи 23.1.8 индукция магнитного поля в области рамки изменяется равномерно, скорость ее изменения постоянна, величина ЭДС индукции не изменяется в процессе проведения опыта (ответ 3).

В задаче 23.1.9 ЭДС индукции, возникающая в рамке во втором случае, вчетверо больше ЭДС индукции, возникающей в первом (ответ 4). Это связано с четырехкратным увеличением площади рамки и, соответственно, магнитного потока через нее во втором случае.

В задаче 23.1.10 во втором случае в два раза увеличивается скорость изменения магнитного потока (индукция поля меняется на ту же величину, но за вдвое меньшее время). Поэтому ЭДС электромагнитной индукции, возникающая в рамке во втором случае, в два раза больше, чем в первом (ответ 1).

При увеличении тока в замкнутом проводнике в два раза (задача 23.2.1), величина индукции магнитного поля возрастет в каждой точке пространства в два раза, не изменившись по направлению. Поэтому ровно в два раза изменится магнитный поток через любую малую площадку и, соответственно, и весь проводник (ответ 1). А вот отношение магнитного потока через проводник к току в этом проводнике, которое и представляет собой индуктивность проводника , при этом не изменится (задача 23.2.2 — ответ 3).

Используя формулу (23.3) находим в задаче 32.2.3 Гн (ответ 4).

Связь между единицами измерений магнитного потока, магнитной индукции и индуктивности (задача 23.2.4) следует из определения индуктивности (23.3): единица магнитного потока (Вб) равна произведению единицы тока (А) на единицу индуктивности (Гн) — ответ 3.

Согласно формуле (23.5) при двукратном увеличении индуктивности катушки и двукратном уменьшении тока в ней (задача 23.2.5) энергия магнитного поля катушки уменьшится в 2 раза (ответ 2).

Когда рамка вращается в однородном магнитном поле, магнитный поток через рамку меняется из-за изменения угла между перпендикуляром к плоскости рамки и вектором индукции магнитного поля. А поскольку и в первом и втором случае в задаче 23.2.6 этот угол меняется по одному и тому же закону (по условию частота вращения рамок одинакова), то ЭДС индукции меняются по одному и тому же закону, и, следовательно, отношение амплитудных значений ЭДС индукции в рамках равно единице (ответ 2).

Магнитное поле, создаваемое проводником с током в области рамки (задача 23.2.7), направлено «от нас» (см. решение задач главы 22). Величина индукции поля провода в области рамки при ее удалении от провода будет уменьшаться. Поэтому индукционный ток в рамке должен создать магнитное поле, направленное внутри рамки «от нас». Используя теперь правило буравчика для нахождения направления магнитной индукции, заключаем, что индукционный ток в рамке будет направлен по часовой стрелке (ответ 1).

При увеличении тока в проводе будет возрастать созданное им магнитное поле и в рамке возникнет индукционный ток (задача 23.2.8). В результате возникнет взаимодействие индукционного тока в рамке и тока в проводнике. Чтобы найти направление этого взаимодействия (притяжение или отталкивание) можно найти направление индукционного тока, а затем по формуле Ампера силу взаимодействия рамки с проводом. Но можно поступить и по-другому, используя правило Ленца. Все индукционные явления должны иметь такое направление, чтобы компенсировать вызывающую их причину. А поскольку причина — увеличение тока в рамке, сила взаимодействия индукционного тока и провода должна стремиться уменьшить магнитный поток поля провода через рамку. А поскольку магнитная индукция поля провода убывает с увеличением расстояния до него, то эта сила будет отталкивать рамку от провода (ответ 2). Если бы ток в проводе убывал, то рамка притягивалась бы к проводу.

Задача 23.2.9 также связана с направлением индукционных явлений и правилом Ленца. При приближении магнита к проводящему кольцу в нем возникнет индукционный ток, причем направление его будет таким, чтобы компенсировать вызывающую его причину. А поскольку эта причина — приближение магнита, кольцо будет отталкиваться от него (ответ 2). Если магнит отодвигать от кольца, то по тем же причинам возникло бы притяжение кольца к магниту.

Задача 23.2.10 — единственная вычислительная задача в этой главе. Для нахождения ЭДС индукции нужно найти изменение магнитного потока через контур . Это можно сделать так. Пусть в некоторый момент времени перемычка находилась в положении, показанном на рисунке, и пусть прошел малый интервал времени . За этот интервал времени перемычка переместится на величину . Это приведет к увеличению площади контура на величину . Поэтому изменение магнитного потока через контур будет равно , а величина ЭДС индукции (ответ 4).

Сила тока. Амперметр — урок. Физика, 8 класс.

В процессе своего движения вдоль проводника заряженные частицы (в металлах это электроны) переносят некоторый заряд. Чем больше заряженных частиц, чем быстрее они движутся, тем больший заряд будет ими перенесён за одно и то же время. Электрический заряд, проходящий через поперечное сечение проводника за 1 секунду, определяет силу тока в цепи.

Сила тока \(I\) — скалярная величина, равная отношению заряда \(q\), прошедшего через поперечное сечение проводника, к промежутку времени \(t\), в течение которого шёл ток.
I=qt, где \(I\) — сила тока, \(q\) — заряд, \(t\) — время.
Единица измерения силы тока в системе СИ — \([I]~=~1~A\) (ампер).

В 1948 г. было предложено в основу определения единицы силы тока положить явление взаимодействия двух проводников с током:


при прохождении тока по двум параллельным проводникам в одном направлении проводники притягиваются, а при прохождении тока по этим же проводникам в противоположных направлениях — отталкиваются.


 

За единицу силы тока \(1~A\) принимают силу тока, при которой два параллельных проводника длиной \(1\) м, расположенные на расстоянии \(1\) м друг от друга в вакууме, взаимодействуют с силой \(0,0000002\)\(H\).

Единица силы тока называется ампером (\(A\)) в честь французского учёного А.-М. Ампера.

 

Андре-Мари Ампер

(1775 — 1836)

 

А.-М. Ампер ввёл термины: электростатика, электродинамика, соленоид, ЭДС, напряжение, гальванометр, электрический ток.


Ампер — довольно большая сила тока. Например, в электрической сети квартиры через включённую \(100\) Вт лампочку накаливания проходит ток с силой, приблизительно равной \(0,5A\). Ток в электрическом обогревателе может достигать \(10A\), а для работы карманного микрокалькулятора достаточно \(0,001A\).

Помимо ампера на практике часто применяются и другие (кратные и дольные) единицы силы тока, например, миллиампер (мА) и микроампер (мкА):
\(1 мA = 0,001 A\), \(1 мкA = 0,000001 A\), \(1 кA =1000 A\).
То есть \(1 A = 1000 мA\), \(1 A = 1000000 мкA\), \(1 A = 0,001 кA\).

Если электроны перемещаются в одном направлении, т.е. — от одного полюса источника тока к другому, то такой ток называют постоянным.

Переменным называется ток, сила и направление которого периодически изменяются.

В бытовых электросетях используют переменный ток напряжением \(220\) В и частотой \(50\) Гц. Это означает, что ток за \(1\) секунду \(50\) раз движется в одном направлении и \(50\) раз — в другом. У многих приборов имеется блок питания, который преобразует переменный ток в постоянный (у телевизора, компьютера и т.д.).

 

Силу тока измеряют амперметром. В электрической цепи он обозначается так:

Амперметр включают в цепь последовательно с тем прибором, силу тока в котором нужно измерить.

Обрати внимание!

Амперметр нельзя подсоединять к источнику тока, если в цепь не подключён потребитель!

Измеряемая сила тока не должна превышать максимально допустимую силу тока для измерения амперметром. Поэтому существуют различные амперметры.

 

Микроамперметр

Миллиамперметр

Амперметр

Килоамперметр

 

Различают амперметры для измерения силы постоянного тока и силы переменного тока.

Обозначения диапазона измерения амперметров:

  • «\(~\)» означает, что амперметр предназначен для измерения силы переменного тока; 
  • «\(—\)» означает, что амперметр предназначен для измерения силы постоянного тока.

Можно обратить внимание на клеммы прибора. Если указана полярность («\(+\)» и «\(-\)»), то это прибор для измерения постоянного тока.

Иногда используют буквы \(AC/DC\). В переводе с английского \(AC\) (alternating current) — переменный ток, а \(DC\) (direct current) — постоянный ток.

Для измерения силы постоянного тока

Для измерения силы переменного тока

 

Для измерения силы тока можно использовать и мультиметр. Перед измерением необходимо прочитать инструкцию, чтобы правильно подключить прибор.

 

Включая амперметр в цепь постоянного тока, необходимо соблюдать полярность (см.схему):
  • провод, который идёт от положительного полюса источника тока, нужно соединять с клеммой амперметра со знаком «\(+\)»;

  • провод, который идёт от отрицательного полюса источника тока, нужно соединять с клеммой амперметра со знаком «\(-\)».

Если полярность на источнике тока не указана, следует помнить, что длинная линия соответствует плюсу, а короткая — минусу.

 

В цепь переменного тока включается амперметр для измерения переменного тока. Он полярности не имеет.

 

Амперметр подключается последовательно к тому прибору, на котором измеряется сила тока.

Значение силы тока до прохождения через лампу равно значению силы тока после прохождения через лампу, значит по всей длине электрической цепи сила тока постоянна (при последовательном соединении).

Обрати внимание!

Безопасным для организма человека можно считать переменный ток силой не выше \(0,05~A\), ток силой более \(0,05\)-\(0,1~A\) опасен и может вызвать смертельный исход.

Источники:

http://class-fizika.narod.ru/8_28.htm
http://school.xvatit.com/index.php?title=%D0%A1%D0%B8%D0%BB%D0%B0_%D1%82%D0%BE%D0%BA%D0%B0
http://physics.kgsu.ru/index.php?option=com_content&view=article&id=217&Itemid=72

http://kamenskih3.narod.ru/untitled74.htm

 

Закон ома — формулировка простыми словами, определение,

Сопротивление

Представьте, что есть труба, в которую затолкали камни. Вода, которая протекает по этой трубе, станет течь медленнее, потому что у нее появилось сопротивление. Точно также будет происходить с электрическим током.

  • Сопротивление — физическая величина, которая показывает способность проводника пропускать электрический ток. Чем выше сопротивление, тем ниже эта способность.

Теперь сделаем «каменный участок» длиннее, то есть добавим еще камней. Воде будет еще сложнее течь.

Сделаем трубу шире, оставив количество камней тем же — воде полегчает, поток увеличится.

Теперь заменим шероховатые камни, которые мы набрали на стройке, на гладкие камушки из моря. Через них проходить тоже легче, а значит сопротивление уменьшается.

Электрический ток реагирует на эти параметры аналогичным образом: при удлинении проводника сопротивление увеличивается, при увеличении поперечного сечения (ширины) проводника сопротивление уменьшается, а если заменить материал — изменится в зависимости от материала.2.

Знайте!

СИ — международная система единиц. «Перевести в СИ» означает перевод всех величин в метры, килограммы, секунды и другие единицы измерения без приставок. Исключение составляет килограмм с приставкой «кило».

  • Удельное сопротивление проводника — это физическая величина, которая показывает способность материала пропускать электрический ток. Это табличная величина, она зависит только от материала.

Таблица удельных сопротивлений различных материалов

Удельное сопротивление

ρ, Ом*мм2/м

Удельное сопротивление

ρ, Ом*мм2/м

Алюминий

0,028

Бронза

0,095 — 0,1

Висмут

1,2

Вольфрам

0,05

Железо

0,1

Золото

0,023

Иридий

0,0474

Константан ( сплав Ni-Cu + Mn)

0,5

Латунь

0,025 — 0,108

Магний

0,045

Манганин (сплав меди марганца и никеля — приборный)

0,43 — 0,51

Медь

0,0175

Молибден

0,059

Нейзильбер (сплав меди цинка и никеля)

0,2

Натрий

0,047

Никелин ( сплав меди и никеля)

0,42

Никель

0,087

Нихром ( сплав никеля хрома железы и марганца)

1,05 — 1,4

Олово

0,12

Платина

0.107

Ртуть

0,94

Свинец

0,22

Серебро

0,015

Сталь

0,103 — 0,137

Титан

0,6

Хромаль

1,3 — 1,5

Цинк

0,054

Чугун

0,5-1,0

Резистор

Все реальные проводники имеют сопротивление, но его стараются сделать незначительным. В задачах вообще используют словосочетание «идеальный проводник», а значит лишают его сопротивления.

Из-за того, что проводник у нас «кругом-бегом-такой-идеальный», чаще всего за сопротивление в цепи отвечает резистор. Это устройство, которое нагружает цепь сопротивлением.

Вот так резистор изображается на схемах:


В школьном курсе физики используют Европейское обозначение, поэтому запоминаем только его. Американское обозначение можно встретить, например, в программе Micro-Cap, в которой инженеры моделируют схемы.

Вот так резистор выглядит в естественной среде обитания:


Полосочки на нем показывают его сопротивление.

На сайте компании Ekits, которая занимается продажей электронных модулей, можно выбрать цвет резистора и узнать значение его сопротивления:


Источник: сайт компании Ekits

О том, зачем дополнительно нагружать сопротивлением цепь, мы поговорим в этой же статье чуть позже.

Не сопротивляйтесь зову сердца и запишите ребенка в современную школу Skysmart. Здесь школьники решают захватывающие задачки по физике и понимают, как это пригодится в жизни.

А еще следят за прогрессом в личном кабинете, задают учителям любые — даже самые неловкие — вопросы и чувствуют себя увереннее на школьных экзаменах и контрольных.2/м]

Закон Ома для участка цепи

С камушками в трубе все понятно, но не только же от них зависит сила, с которой поток воды идет по трубе — от насоса, которым мы эту воду качаем, тоже зависит. Чем сильнее качаем, тем больше течение. В электрической цепи функцию насоса выполняет источник тока.

Например, источником может быть гальванический элемент (привычная батарейка). Батарейка работает на основе химических реакций внутри нее. Эти реакции выделяют энергию, которая потом передается электрической цепи.

У любого источника обязательно есть полюса — «плюс» и «минус». Полюса — это его крайние положения, по сути клеммы, к которым присоединяется электрическая цепь. Собственно, ток как раз течет от «+» к «-».


У нас уже есть две величины, от которых зависит электрический ток в цепи — напряжение и сопротивление. Кажется, пора объединять их в закон.

Сила тока в участке цепи прямо пропорциональна напряжению на его концах и обратно пропорциональна его сопротивлению.

Математически его можно описать вот так:

Закон Ома для участка цепи

I = U/R

I — сила тока [A]

U — напряжение [В]

R — сопротивление [Ом]

Напряжение измеряется в Вольтах и показывает разницу между двумя точками цепи: от этой разницы зависит, насколько сильно будет течь ток — чем больше разница, тем выше напряжение и ток будет течь сильнее.

Сила тока измеряется в Амперах, а подробнее о ней вы можете прочитать в нашей статье 😇

Давайте решим несколько задач на Закон Ома для участка цепи.

Задача раз

Найти силу тока в лампочке накаливания, если торшер включили в сеть напряжением 220 В, а сопротивление нити накаливания равно 880 Ом.2/м

Обратимся к таблице удельных сопротивлений материалов, чтобы выяснить, из какого материала сделана эта нить накаливания.

Таблица удельных сопротивлений различных материалов

Удельное сопротивление

ρ, Ом*мм2/м

Удельное сопротивление

ρ, Ом*мм2/м

Алюминий

0,028

Бронза

0,095 — 0,1

Висмут

1,2

Вольфрам

0,05

Железо

0,1

Золото

0,023

Иридий

0,0474

Константан ( сплав Ni-Cu + Mn)

0,5

Латунь

0,025 — 0,108

Магний

0,045

Манганин (сплав меди марганца и никеля — приборный)

0,43 — 0,51

Медь

0,0175

Молибден

0,059

Нейзильбер (сплав меди цинка и никеля)

0,2

Натрий

0,047

Никелин ( сплав меди и никеля)

0,42

Никель

0,087

Нихром ( сплав никеля хрома железы и марганца)

1,05 — 1,4

Олово

0,12

Платина

0.107

Ртуть

0,94

Свинец

0,22

Серебро

0,015

Сталь

0,103 — 0,137

Титан

0,6

Хромаль

1,3 — 1,5

Цинк

0,054

Чугун

0,5-1,0

Ответ: нить накаливания сделана из константана.

Закон Ома для полной цепи

Мы разобрались с законом Ома для участка цепи. А теперь давайте узнаем, что происходит, если цепь полная: у нее есть источник, проводники, резисторы и другие элементы.

В таком случае вводится Закон Ома для полной цепи: сила тока в полной цепи равна отношению ЭДС цепи к ее полному сопротивлению.

Так, стоп. Слишком много незнакомых слов — разбираемся по-порядку.

Что такое ЭДС и откуда она берется

ЭДС расшифровывается, как электродвижущая сила. Обозначается греческой буквой ε и измеряется, как и напряжение, в Вольтах.

  • ЭДС — это сила, которая движет заряженные частицы в цепи. Она берется из источника тока. Например, из батарейки.

Химическая реакция внутри гальванического элемента (это синоним батарейки) происходит с выделением энергии в электрическую цепь. Именно эта энергия заставляет частицы двигаться по проводнику.

Зачастую напряжение и ЭДС приравнивают и говорят, что это одно и то же. Формально, это не так, но при решении задач чаще всего и правда нет разницы, так как эти величины обе измеряются в Вольтах и определяют очень похожие по сути своей процессы.

В виде формулы Закон Ома для полной цепи будет выглядеть следующим образом:

Закон Ома для полной цепи

I = ε/(R + r)

I — сила тока [A]

ε — ЭДС [В]

R — сопротивление [Ом]

r — внутреннее сопротивление источника [Ом]

Любой источник не идеален. В задачах это возможно («источник считать идеальным», вот эти вот фразочки), но в реальной жизни — точно нет. В связи с этим у источника есть внутреннее сопротивление, которое мешает протеканию тока.

Решим задачу на полную цепь.

Задачка

Найти силу тока в полной цепи, состоящей из одного резистора сопротивлением 3 Ом и источником с ЭДС равной 4 В и внутренним сопротивлением 1 Ом

Решение:

Возьмем закон Ома для полной цепи:

I = ε/(R + r)

Подставим значения:

I = 4/(3+1) = 1 A

Ответ: сила тока в цепи равна 1 А.

Когда «сопротивление бесполезно»

Электрический ток — умный и хитрый парень. Если у него есть возможность обойти резистор и пойти по идеальному проводнику без сопротивления, он это сделает. При этом с резисторами просто разных номиналов это не сработает: он не пойдет просто через меньшее сопротивление, а распределится согласно закону Ома — больше тока пойдет туда, где сопротивление меньше, и наоборот.

А вот на рисунке ниже сопротивление цепи равно нулю, потому что ток через резистор не пойдет.


Ток идет по пути наименьшего сопротивления.

Теперь давайте посмотрим на закон Ома для участка цепи еще раз.

Закон Ома для участка цепи

I = U/R

I — сила тока [A]

U — напряжение [В]

R — сопротивление [Ом]

Подставим сопротивление, равное 0. Получается, что знаменатель равен нулю, а на математике говорят, что на ноль делить нельзя. Но мы вам раскроем страшную тайну, только не говорите математикам: на ноль делить можно. Если совсем упрощать такое сложное вычисление (а именно потому что оно сложное, мы всегда говорим, что его нельзя производить), то получится бесконечность.

То есть:

I = U/0 = ∞

Такой случай называют коротким замыканием — когда величина силы тока настолько велика, что можно устремить ее к бесконечности. В таких ситуациях мы видим искру, бурю, безумие — и все ломается.

Это происходит, потому что две точки цепи имеют между собой напряжение (то есть между ними есть разница). Это как если вдоль реки неожиданно появляется водопад. Из-за этой разницы возникает искра, которую можно избежать, поставив в цепь резистор.

Именно во избежание коротких замыканий нужно дополнительное сопротивление в цепи.

Параллельное и последовательное соединение

Все это время речь шла о цепях с одним резистором. Рассмотрим, что происходит, если их больше.


Последовательное соединение

Параллельное соединение

Схема

Резисторы следуют друг за другом

Между резисторами есть два узла

Узел — это соединение трех и более проводников

Сила тока

Сила тока одинакова на всех резисторах

I = I1 = I2

Сила тока, входящего в узел, равна сумме сил токов, выходящих из него

I = I1 + I2

Напряжение

Общее напряжение цепи складывается из напряжений на каждом резисторе

U = U1 + U2

Напряжение одинаково на всех резисторах

U = U1 = U2

Сопротивление

Общее сопротивление цепи складывается из сопротивлений каждого резистора

R = R1 + R2

Общее сопротивление для бесконечного количества параллельно соединенных резисторов

1/R = 1/R1 + 1/R2 + … + 1/Rn

Общее сопротивление для двух параллельно соединенных резисторов

R = (R1 * R2)/R1 + R2

Общее сопротивление бесконечного количества параллельно соединенных одинаковых резисторов

R = R1/n

Зачем нужны эти соединения, если можно сразу взять резистор нужного номинала?

Начнем с того, что все электронные компоненты изготавливаются по ГОСТу. То есть есть определенные значения резисторов, от которых нельзя отойти при производстве. Это значит, что не всегда есть резистор нужного номинала и его нужно соорудить из других резисторов.

Параллельное соединение также используют, как «запасной аэродром»: когда на конечный результат общее сопротивление сильно не повлияет, но в случае отказа одного из резисторов, будет работать другой.

Признаемся честно: схемы, которые обычно дают в задачах (миллион параллельно соединенных резисторов, к ним еще последовательный, а к этому последовательному еще миллион параллельных) — в жизни не встречаются. Но навык расчета таких схем впоследствии упрощает подсчет схем реальных, потому что так вы невооруженным глазом отличаете последовательное соединение от параллельного.

Решим несколько задач на последовательное и параллельное соединение.

Задачка раз

Найти общее сопротивление цепи.

R1 = 1 Ом, R2 = 2 Ом, R3 = 3 Ом, R4 = 4 Ом.


Решение:

Общее сопротивление при последовательном соединении рассчитывается по формуле:

R = R1 + R2 + R3 + R4 = 1 + 2 + 3 + 4 = 10 Ом

Ответ: общее сопротивление цепи равно 10 Ом

Задачка два

Найти общее сопротивление цепи.

R1 = 4 Ом, R2 = 2 Ом


Решение:

Общее сопротивление при параллельном соединении рассчитывается по формуле:

R = (R1 * R2)/R1 + R2 = 4*2/4+2 = 4/3 = 1 ⅓ Ом

Ответ: общее сопротивление цепи равно 1 ⅓ Ом

Задачка три

Найти общее сопротивление цепи, состоящей из резистора и двух ламп.

R1 = 1 Ом, R2 = 2 Ом, R3 = 3 Ом


Решение:

Сначала обозначим, что лампы с точки зрения элемента электрической цепи не отличаются от резисторов. То есть у них тоже есть сопротивление, и они также влияют на цепь.

В данном случае соединение является смешанным. Лампы соеденены параллельно, а последовательно к ним подключен резистор.

Сначала посчитаем общее сопротивление для ламп. Общее сопротивление при параллельном соединении рассчитывается по формуле:

Rламп = (R2 * R3)/R2 + R3 = 2*3/2+3 = 6/5 = 1,2 Ом

Общее сопротивление при последовательном соединении рассчитывается по формуле:

R = R1 + Rламп = 1 + 1,2 = 2,2 Ом

Ответ: общее сопротивление цепи равно 2,2 Ом.

Наконец-то, последняя и самая сложная задача! В ней собрали все самое серьезное из этой статьи 💪.

Задачка четыре со звездочкой

К аккумулятору с ЭДС 12 В, подключена лампочка и два параллельно соединенных резистора сопротивлением каждый по 10 Ом. Известно, что ток в цепи 0,5 А, а сопротивление лампочки R/2.2)/2R = R/2 = 10/2 = 5 Ом

И общее сопротивление цепи равно:

R = Rлампы + Rрезисторов = 5 + 5 = 10 Ом

Выразим внутреннее сопротивление источника из закона Ома для полной цепи.

I = ε/(R + r)

R + r = ε/I

r = ε/I — R

Подставим значения:

r = 12/0,5 — 10 = 14 Ом

Ответ: внутреннее сопротивление источника равно 14 Ом.

Чтобы ребенок научился решать самые сложные задачи и чувствовал себя уверенно на олимпиадах и экзаменах, запишите его на бесплатный вводный урок в Skysmart.

Профессиональные учителя физики не только научат решать задачи и подготовят к экзамену, но и объяснят, как это все устроено: легко, интерактивно и с примерами из реальной жизни современных подростков.

10.2: Электродвижущая сила — Physics LibreTexts

Цели обучения

К концу раздела вы сможете:

  • Опишите электродвижущую силу (ЭДС) и внутреннее сопротивление батареи
  • Объясните основную работу аккумулятора

Если вы забудете выключить автомобильные фары, они будут постепенно тускнеть по мере разрядки аккумулятора. Почему они не мигают внезапно, когда разрядился аккумулятор? Их постепенное затемнение означает, что выходное напряжение батареи уменьшается по мере разряда батареи.Причина снижения выходного напряжения для разряженных батарей заключается в том, что все источники напряжения состоят из двух основных частей — источника электрической энергии и внутреннего сопротивления. В этом разделе мы исследуем источник энергии и внутреннее сопротивление.

Введение в электродвижущей силы

Voltage имеет множество источников, некоторые из которых показаны на рисунке \ (\ PageIndex {2} \). Все такие устройства создают разность потенциалов и могут подавать ток, если подключены к цепи.Особый тип разности потенциалов известен как электродвижущая сила (ЭДС) . ЭДС — это вовсе не сила, но термин «электродвижущая сила» используется по историческим причинам. Он был изобретен Алессандро Вольта в 1800-х годах, когда он изобрел первую батарею, также известную как гальваническая батарея . Поскольку электродвижущая сила не является силой, принято называть эти источники просто источниками ЭДС (произносимыми буквами «ee-em-eff»), а не источниками электродвижущей силы.

Рисунок \ (\ PageIndex {1} \): различные источники напряжения. а) ветряная электростанция Бразос в Флуванна, штат Техас; (б) Красноярская плотина в России; (c) солнечная ферма; (d) группа никель-металлогидридных батарей. Выходное напряжение каждого устройства зависит от его конструкции и нагрузки. Выходное напряжение равно ЭДС только при отсутствии нагрузки. (кредит a: модификация работы «Leaflet» / Wikimedia Commons; кредит b: модификация работы Алекса Полежаева; кредит c: модификация работы Министерства энергетики США; кредит d: модификация работы Тиаа Монто)

Если Электродвижущая сила — это вообще не сила, тогда что такое ЭДС и что является источником ЭДС? Чтобы ответить на эти вопросы, рассмотрим простую схему лампы 12 В, подключенной к батарее 12 В, как показано на рисунке \ (\ PageIndex {2} \).Батарея , может быть смоделирована как устройство с двумя выводами, которое поддерживает один вывод с более высоким электрическим потенциалом, чем второй вывод. Более высокий электрический потенциал иногда называют положительной клеммой и обозначают знаком плюс. Клемму с более низким потенциалом иногда называют отрицательной клеммой и обозначают знаком минус. Это источник ЭДС.

Рисунок \ (\ PageIndex {2} \): Источник ЭДС поддерживает на одном выводе более высокий электрический потенциал, чем на другом выводе, действуя как источник тока в цепи.

Когда источник ЭДС не подключен к лампе, нет чистого потока заряда внутри источника ЭДС. Как только батарея подключена к лампе, заряды перетекают от одной клеммы батареи через лампу (в результате чего лампа загорается) и обратно к другой клемме батареи. Если мы рассмотрим протекание положительного (обычного) тока, положительные заряды покидают положительный вывод, проходят через лампу и попадают в отрицательный вывод.

Положительный поток тока полезен для большей части анализа схем в этой главе, но в металлических проводах и резисторах наибольший вклад в ток вносят электроны, протекающие в направлении, противоположном положительному потоку тока.Поэтому более реалистично рассматривать движение электронов для анализа схемы на рисунке \ (\ PageIndex {2} \). Электроны покидают отрицательную клемму, проходят через лампу и возвращаются к положительной клемме. Чтобы источник ЭДС поддерживал разность потенциалов между двумя выводами, отрицательные заряды (электроны) должны быть перемещены с положительного вывода на отрицательный. Источник ЭДС действует как накачка заряда, перемещая отрицательные заряды от положительного вывода к отрицательному для поддержания разности потенциалов.Это увеличивает потенциальную энергию зарядов и, следовательно, электрический потенциал зарядов.

Сила, действующая на отрицательный заряд электрического поля, действует в направлении, противоположном электрическому полю, как показано на рисунке \ (\ PageIndex {2} \). Чтобы отрицательные заряды переместились на отрицательную клемму, необходимо провести работу с отрицательными зарядами. Для этого требуется энергия, которая возникает в результате химических реакций в батарее. Потенциал поддерживается высоким на положительной клемме и низким на отрицательной клемме, чтобы поддерживать разность потенциалов между двумя клеммами.ЭДС равна работе, выполняемой над зарядом на единицу заряда \ (\ left (\ epsilon = \ frac {dW} {dq} \ right) \) при отсутствии тока. Поскольку единицей работы является джоуль, а единицей заряда — кулон, единицей измерения ЭДС является вольт \ ((1 \, V = 1 \, J / C) \).

Напряжение на клеммах \ (V_ {клемма} \) батареи — это напряжение, измеренное на клеммах батареи, когда к клемме не подключена нагрузка. Идеальная батарея — это источник ЭДС, который поддерживает постоянное напряжение на клеммах, независимо от тока между двумя клеммами.Идеальная батарея не имеет внутреннего сопротивления, а напряжение на клеммах равно ЭДС батареи. В следующем разделе мы покажем, что у реальной батареи есть внутреннее сопротивление, а напряжение на клеммах всегда меньше, чем ЭДС батареи.

Происхождение потенциала батареи

ЭДС батареи определяется сочетанием химических веществ и составом выводов батареи. Свинцово-кислотный аккумулятор , используемый в автомобилях и других транспортных средствах, является одним из наиболее распространенных сочетаний химических веществ.На рисунке \ (\ PageIndex {3} \) показана одна ячейка (одна из шести) этой батареи. Катодная (положительная) клемма ячейки соединена с пластиной из оксида свинца, а анодная (отрицательная) клемма подключена к свинцовой пластине. Обе пластины погружены в серную кислоту, электролит для системы.

Рисунок \ (\ PageIndex {3} \): Химические реакции в свинцово-кислотном элементе разделяют заряд, отправляя отрицательный заряд на анод, который соединен со свинцовыми пластинами. Пластины из оксида свинца подключаются к положительному или катодному выводу ячейки.Серная кислота проводит заряд, а также участвует в химической реакции.

Небольшое знание того, как взаимодействуют химические вещества в свинцово-кислотной батарее, помогает понять потенциал, создаваемый батареей. На рисунке \ (\ PageIndex {4} \) показан результат одной химической реакции. Два электрона помещаются на анод , что делает его отрицательным, при условии, что катод подает два электрона. Это оставляет катод положительно заряженным, потому что он потерял два электрона.Короче говоря, разделение заряда было вызвано химической реакцией.

Обратите внимание, что реакция не происходит, если нет полной цепи, позволяющей подавать два электрона на катод. Во многих случаях эти электроны выходят из анода, проходят через сопротивление и возвращаются на катод. Отметим также, что, поскольку в химических реакциях участвуют вещества, обладающие сопротивлением, невозможно создать ЭДС без внутреннего сопротивления.

Рисунок \ (\ PageIndex {4} \): В свинцово-кислотной батарее два электрона прижимаются к аноду элемента, а два электрона удаляются с катода элемента.В результате химической реакции в свинцово-кислотной батарее два электрона помещаются на анод и два электрона удаляются с катода. Для работы требуется замкнутая цепь, так как два электрона должны быть доставлены на катод.

Внутреннее сопротивление и напряжение на клеммах

Величина сопротивления прохождению тока внутри источника напряжения называется внутренним сопротивлением . Внутреннее сопротивление батареи r может вести себя сложным образом. Обычно она увеличивается по мере разряда батареи из-за окисления пластин или снижения кислотности электролита.Однако внутреннее сопротивление также может зависеть от величины и направления тока через источник напряжения, его температуры и даже его предыстории. Например, внутреннее сопротивление перезаряжаемых никель-кадмиевых элементов зависит от того, сколько раз и насколько глубоко они были разряжены. Простая модель батареи состоит из идеализированного источника ЭДС \ (\ epsilon \) и внутреннего сопротивления r (рисунок \ (\ PageIndex {5} \)).

Рисунок \ (\ PageIndex {5} \): Батарею можно смоделировать как идеализированную ЭДС \ ((\ epsilon) \) с внутренним сопротивлением ( r ).Напряжение на клеммах аккумулятора равно \ (V_ {terminal} = \ epsilon — Ir \).

Предположим, что внешний резистор, известный как сопротивление нагрузки R , подключен к источнику напряжения, например батарее, как показано на рисунке \ (\ PageIndex {6} \). На рисунке показана модель аккумулятора с ЭДС ε, внутренним сопротивлением r и нагрузочным резистором R , подключенным к его клеммам. При обычном протекании тока положительные заряды покидают положительную клемму батареи, проходят через резистор и возвращаются к отрицательной клемме батареи.Напряжение на клеммах аккумулятора зависит от ЭДС, внутреннего сопротивления и силы тока и равно

.

Примечание

\ [V_ {терминал} = \ epsilon — Ir \]

При заданной ЭДС и внутреннем сопротивлении напряжение на клеммах уменьшается по мере увеличения тока из-за падения потенциала Ir внутреннего сопротивления.

Рисунок \ (\ PageIndex {6} \): Схема источника напряжения и его нагрузочного резистора R . Поскольку внутреннее сопротивление r последовательно с нагрузкой, оно может значительно повлиять на напряжение на клеммах и ток, подаваемый на нагрузку.

График разности потенциалов на каждом элементе цепи показан на рисунке \ (\ PageIndex {7} \). Через цепь проходит ток I , а падение потенциала на внутреннем резисторе равно Ir . Напряжение на клеммах равно \ (\ epsilon — Ir \), что равно падению потенциала на нагрузочном резисторе \ (IR = \ epsilon — Ir \). Как и в случае с потенциальной энергией, важно изменение напряжения. Когда используется термин «напряжение», мы предполагаем, что это на самом деле изменение потенциала, или \ (\ Delta V \).Однако \ (\ Delta \) часто для удобства опускается.

Рисунок \ (\ PageIndex {7} \): график напряжения в цепи батареи и сопротивления нагрузки. Электрический потенциал увеличивает ЭДС батареи из-за химических реакций, выполняющих работу с зарядами. В аккумуляторе происходит снижение электрического потенциала из-за внутреннего сопротивления. Потенциал уменьшается из-за внутреннего сопротивления \ (- Ir \), в результате чего напряжение на клеммах батареи равно \ ((\ epsilon — Ir) \).Затем напряжение уменьшается на ( IR ). Ток равен \ (I = \ frac {\ epsilon} {r + R} \).

Ток через нагрузочный резистор равен \ (I = \ frac {\ epsilon} {r + R} \). Из этого выражения видно, что чем меньше внутреннее сопротивление r , тем больший ток подает источник напряжения на свою нагрузку R . По мере разряда батарей r увеличивается. Если r становится значительной частью сопротивления нагрузки, то ток значительно снижается, как показано в следующем примере.

Пример \ (\ PageIndex {1} \): анализ цепи с батареей и нагрузкой

Данная батарея имеет ЭДС 12,00 В и внутреннее сопротивление \ (0,100 \, \ Омега \). (a) Рассчитайте напряжение на его клеммах при подключении к нагрузке с \ (10.00 \, \ Omega \). (b) Какое напряжение на клеммах при подключении к нагрузке \ (0.500 \, \ Omega \)? (c) Какая мощность рассеивается при нагрузке \ (0.500 \, \ Omega \)? (d) Если внутреннее сопротивление увеличивается до \ (0.500 \, \ Omega \), найдите ток, напряжение на клеммах и мощность, рассеиваемую \ (0.500 \, \ Omega \) загрузка.

Стратегия

Приведенный выше анализ дал выражение для тока с учетом внутреннего сопротивления. Как только ток будет найден, напряжение на клеммах можно рассчитать с помощью уравнения \ (V_ {terminal} = \ epsilon — Ir \). Как только ток будет найден, мы также сможем найти мощность, рассеиваемую резистором.

Решение

  1. Ввод заданных значений ЭДС, сопротивления нагрузки и внутреннего сопротивления в выражение выше дает \ [I = \ frac {\ epsilon} {R + r} = \ frac {12.00 \, V} {10.10 \, \ Omega} = 1.188 \, A. \] Введите известные значения в уравнение \ (V_ {terminal} = \ epsilon — Ir \), чтобы получить напряжение на клеммах: \ [V_ { клемма} = \ epsilon — Ir = 12.00 \, V — (1.188 \, A) (0.100 \, \ Omega) = 11.90 \, V. \] Напряжение на клеммах здесь лишь немного ниже, чем ЭДС, что означает, что ток втягивается этой легкой нагрузкой незначительно.
  2. Аналогично, при \ (R_ {load} = 0.500 \, \ Omega \) ток равен \ [I = \ frac {\ epsilon} {R + r} = \ frac {12.00 \, V} {0.2} {R} \) или \ (IV \), где В, — напряжение на клеммах (в данном случае 10,0 В).
  3. Здесь внутреннее сопротивление увеличилось, возможно, из-за разряда батареи, до точки, где оно равно сопротивлению нагрузки. Как и раньше, мы сначала находим ток, вводя известные значения в выражение, получая \ [I = \ frac {\ epsilon} {R + r} = \ frac {12.00 \, V} {1.00 \, \ Omega} = 12.00 \, A. \] Теперь напряжение на клеммах равно \ [V_ {terminal} = \ epsilon — Ir = 12.00 \, V — (12.2 (0.500 \, \ Omega) = 72.00 \, W. \] Мы видим, что повышенное внутреннее сопротивление значительно снизило напряжение на клеммах, ток и мощность, подаваемую на нагрузку.

Значение

Внутреннее сопротивление батареи может увеличиваться по многим причинам. Например, внутреннее сопротивление перезаряжаемой батареи увеличивается с увеличением количества раз, когда батарея перезаряжается. Повышенное внутреннее сопротивление может иметь двоякое влияние на аккумулятор.Сначала снизится напряжение на клеммах. Во-вторых, аккумулятор может перегреться из-за повышенной мощности, рассеиваемой внутренним сопротивлением.

Упражнение \ (\ PageIndex {1} \)

Если вы поместите провод прямо между двумя выводами батареи, эффективно закоротив клеммы, батарея начнет нагреваться. Как вы думаете, почему это происходит?

Решение

Если к клеммам подключен провод, сопротивление нагрузки близко к нулю или, по крайней мере, значительно меньше внутреннего сопротивления батареи.2р) \). Мощность рассеивается в виде тепла.

Тестеры батарей

Тестеры батарей, такие как те, что показаны на рисунке \ (\ PageIndex {8} \), используют малые нагрузочные резисторы, чтобы намеренно потреблять ток, чтобы определить, падает ли потенциал клемм ниже допустимого уровня. Хотя измерить внутреннее сопротивление батареи сложно, тестеры батареи могут обеспечить измерение внутреннего сопротивления батареи. Если внутреннее сопротивление высокое, батарея разряжена, о чем свидетельствует низкое напряжение на клеммах.

Рисунок \ (\ PageIndex {8} \): Тестеры батарей измеряют напряжение на клеммах под нагрузкой, чтобы определить состояние батареи. (a) Техник-электронщик ВМС США использует тестер аккумуляторов для проверки больших аккумуляторов на борту авианосца USS Nimitz . Тестер батарей, который она использует, имеет небольшое сопротивление, которое может рассеивать большое количество энергии. (b) Показанное небольшое устройство используется на небольших батареях и имеет цифровой дисплей для индикации допустимого напряжения на клеммах. (кредит А: модификация работы Джейсона А.Джонстон; кредит b: модификация работы Кейта Уильямсона)

Некоторые батареи можно заряжать, пропуская через них ток в направлении, противоположном току, который они подают в прибор. Это обычно делается в автомобилях и батареях для небольших электроприборов и электронных устройств (Рисунок \ (\ PageIndex {9} \)). Выходное напряжение зарядного устройства должно быть больше, чем ЭДС аккумулятора, чтобы ток через него реверсировал. Это приводит к тому, что напряжение на клеммах батареи превышает ЭДС, поскольку \ (V = \ epsilon — Ir \) и I теперь отрицательны.

Рисунок \ (\ PageIndex {9} \): автомобильное зарядное устройство меняет нормальное направление тока через аккумулятор, обращая вспять его химическую реакцию и пополняя ее химический потенциал.

Важно понимать последствия внутреннего сопротивления источников ЭДС, таких как батареи и солнечные элементы, но часто анализ цепей выполняется с помощью напряжения на клеммах батареи, как мы делали в предыдущих разделах. Напряжение на клеммах обозначается просто как В , без индекса «клемма».Это связано с тем, что внутреннее сопротивление батареи трудно измерить напрямую, и оно может со временем измениться.

Авторы и авторство

Сэмюэл Дж. Линг (Государственный университет Трумэна), Джефф Санни (Университет Лойола Мэримаунт) и Билл Мобс со многими авторами. Эта работа лицензирована OpenStax University Physics в соответствии с лицензией Creative Commons Attribution License (4.0).

Как рассчитать ЭДС | Sciencing

Обновлено 2 ноября 2020 г.

Ли Джонсон

Электродвижущая сила (ЭДС) — понятие незнакомое для большинства людей, но оно тесно связано с более знакомым понятием напряжения.Понимание разницы между ними и того, что означает ЭДС, дает вам инструменты, необходимые для решения многих проблем в физике и электронике, а также знакомит с концепцией внутреннего сопротивления батареи. ЭДС сообщает вам напряжение батареи без уменьшения внутреннего сопротивления, как это происходит при обычных измерениях разности потенциалов. Вы можете рассчитать его несколькими способами, в зависимости от того, какая информация у вас есть.

TL; DR (слишком долго; не читал)

Рассчитайте ЭДС по формуле:

ε = V + Ir

Здесь (В) означает напряжение элемента, (I) означает ток в цепи, а (r) означает внутреннее сопротивление ячейки.

Что такое ЭДС?

Электродвижущая сила — это разность потенциалов (т. Е. Напряжение) на клеммах батареи при отсутствии тока. Может показаться, что это не имеет значения, но каждая батарея имеет «внутреннее сопротивление». Это похоже на обычное сопротивление, которое снижает ток в цепи, но оно существует внутри самой батареи. Это связано с тем, что материалы, из которых состоят элементы в батарее, имеют собственное сопротивление (так как практически все материалы имеют).

Когда через элемент не течет ток, это внутреннее сопротивление ничего не меняет, потому что нет тока, который мог бы замедлить его. В некотором смысле, ЭДС можно рассматривать как максимальную разность потенциалов на клеммах в идеальной ситуации, и на практике она всегда больше, чем напряжение батареи.

Уравнения для расчета ЭДС

Есть два основных уравнения для расчета ЭДС. Наиболее фундаментальное определение — это количество джоулей энергии (E), которое набирает каждый кулон заряда (Q) при прохождении через ячейку:

Где (ε) — символ электродвижущей силы, (E) — энергия в цепи, а (Q) — заряд цепи.Если вы знаете результирующую энергию и количество заряда, проходящего через ячейку, это самый простой способ рассчитать ЭДС, но в большинстве случаев у вас нет этой информации.

Вместо этого вы можете использовать определение, больше похожее на закон Ома (V = IR). Это может быть выражено как:

\ epsilon = I (R + r)

, где (I) означает ток, (R) — сопротивление рассматриваемой цепи, а (r) — внутреннее сопротивление ячейки. Расширение этого показывает тесную связь с законом Ома:

\ epsilon = IR + Ir = V + Ir

Это показывает, что вы можете рассчитать ЭДС, если вы знаете напряжение на клеммах (напряжение, используемое в реальных ситуациях) , протекающий ток и внутреннее сопротивление ячейки.

Как рассчитать ЭДС: пример

В качестве примера представьте, что у вас есть цепь с разностью потенциалов 3,2 В, протекающим током 0,6 А и внутренним сопротивлением батареи 0,5 Ом. Используя формулу выше:

\ epsilon = V + Ir = 3.2 \ text {V} + (0.6 \ text {A}) (0.5 \ text {} \ Omega) = 3.5 \ text {V}

Итак, ЭДС этой цепи составляет 3,5 В.

Физика — Электродвижущая сила — Бирмингемский университет

Электродвижущая сила (ЭДС) равна разности потенциалов на клеммах при отсутствии тока.ЭДС и разность потенциалов на клеммах ( В, ) измеряются в вольтах, но это не одно и то же. ЭДС ( ϵ ) — это количество энергии ( E ), обеспечиваемое батареей каждому кулону заряда ( Q ), проходящего через нее.

Как рассчитать ЭДС?

ЭДС можно записать через внутреннее сопротивление батареи ( r ) где: ϵ = I (r + R )

Что из закона Ома, мы можем затем изменить это в терминах оконечного сопротивления: ϵ = В + Ir

ЭДС ячейки может быть определена путем измерения напряжения на ячейке с помощью вольтметра и тока в цепи с помощью амперметра для различных сопротивлений.Затем мы можем настроить схему для определения ЭДС, как показано ниже.

ЭДС и внутреннее сопротивление электрических элементов и батарей

Исследование ЭМП

Как закон Фарадея соотносится с ЭМП?

Закон Фарадея гласит, что любое изменение магнитного поля катушки будет индуцировать в катушке ЭДС (а следовательно, и ток). Он пропорционален минус скорости изменения магнитного потока ( ϕ ) (примечание N — количество витков в катушке).

Согласно закону Фарадея, общество извлекло выгоду из таких важных технологий, как трансформаторы, которые используются для передачи электроэнергии в национальной энергосистеме Великобритании, которая сейчас необходима в наших домах. Также он используется в электрических генераторах и двигателях, таких как плотины гидроэлектростанций, которые производят электричество, которое сейчас является неотъемлемой частью наших современных технологических потребностей. Текущий исследовательский проект MAG-DRIVE в Бирмингеме направлен на поиск способов разработки и улучшения материалов с постоянными магнитами, которые можно использовать в электромобилях следующего поколения.ЭМП также генерируется солнечными батареями, поэтому они важны для исследований в области возобновляемых источников энергии.

Лабораторные признания

Исследователи подкаста In the Laboratory Confessions рассказывают о своем лабораторном опыте в контексте практических экзаменов A Level. Эпизоды, посвященные правильному использованию цифровых инструментов (простое гармоническое движение), правильному построению принципиальных схем (удельное сопротивление в проводе) и использованию источников питания постоянного тока (конденсаторов), имеют отношение к эксперименту по ЭДС, ниже вы можете услышать удельное сопротивление. в проводном подкасте.

Как мы интерпретируем наши данные?

По мере увеличения сопротивления переменного резистора величина тока будет уменьшаться. График зависимости напряжения от тока должен давать линейную зависимость, где градиент линии дает отрицательное внутреннее сопротивление ячейки ( -r ), а точка пересечения дает ЭДС (напряжение, при котором ток равен 0).

Выполнение нескольких измерений при разных значениях сопротивления даст больше точек на графике V-I, что сделает подбор более надежным.Также рекомендуется повторить измерения, так как ячейка будет постепенно стекать, что повлияет на показания. Во избежание разряда элемента / батареи ее следует отключать между измерениями. В качестве альтернативы в схему можно включить выключатель. Также не рекомендуется использовать аккумуляторные батареи, так как они имеют низкое внутреннее сопротивление.

Несмотря на то, что этот эксперимент довольно прост, он поможет вам отличить конечную разницу от ЭДС, что может быть сложной концепцией для понимания учащимися.Поскольку люди все больше полагаются на электричество, исследования, связанные с ЭМП, важны для развития и технического прогресса электричества.

Следующие шаги

Эти ссылки предоставлены только для удобства и в информационных целях; они не означают одобрения или одобрения Бирмингемским университетом какой-либо информации, содержащейся на внешнем веб-сайте. Бирмингемский университет не несет ответственности за точность, законность или содержание внешнего сайта или последующих ссылок.Пожалуйста, свяжитесь с внешним сайтом для получения ответов на вопросы относительно его содержания.

Электродвижущая сила и внутреннее сопротивление

Электродвижущая сила (э) или э.д.с. это энергия, обеспечиваемая элементом или батареей на один кулон заряда, проходящего через них, она равна , измеренная в вольтах (В). Равен разности потенциалов на выводах ячейки при отсутствии тока.

  • e = электродвижущая сила в вольтах, В
  • E = энергия в джоулях, Дж
  • Q = заряд в кулонах, Кл

Батареи и элементы имеют внутреннее сопротивление (r) , которое составляет единиц измерения в омах (Вт). Когда электричество течет по цепи, внутреннее сопротивление самого элемента сопротивляется прохождению тока, и поэтому тепловая энергия теряется в самом элементе.

  • e = электродвижущая сила в вольтах, В
  • I = ток в амперах, А
  • R = сопротивление нагрузки в цепи в Ом, Вт
  • r = внутреннее сопротивление ячейки в Ом, Вт

Мы можем изменить приведенное выше уравнение;

, а затем на

В этом уравнении ( В, ) появляется разность потенциалов , измеренная в вольтах (В).Это разность потенциалов на выводах ячейки при протекании тока в цепи, она всегда меньше ЭДС. ячейки.

Пример;

Q1) p.d. на выводах элемента составляет 3,0 В, когда он не подключен к цепи и не течет ток. Когда ячейка подключена к цепи и течет ток 0,37 А, клемма p.d. падает до 2,8 В. Какое внутреннее сопротивление ячейки?

График терминала п.d. против текущего

Если мы построим график разности потенциалов на клеммах (V) в зависимости от тока в цепи (I), мы получим прямую линию с отрицательным градиентом.

Мы можем им переставить э.д.с. уравнение сверху, чтобы соответствовать общему выражению для прямой линии, y = mx + c.

Из красных прямоугольников выше видно;

  • точка пересечения по оси Y равна э.д.с. ячейки
  • градиент графика равен -r, где r — внутреннее сопротивление ячейки.

Оценка внутреннего сопротивления в цепях | Электрические схемы

Рабочий пример 7: Внутреннее сопротивление в цепи с последовательными резисторами

Для следующей схемы рассчитайте:

  1. разности потенциалов \ (V_ \ text {1} \), \ (V_ \ text {2} \) и \ (V_ \ text {3} \) на резисторах \ (R_ \ text {1} \), \ (R_ \ text {2} \) и \ (R_ \ text {3} \)

    .
  2. сопротивление \ (R_ \ text {3} \).

  3. сопротивление \ (R_ \ text {3} \).

Если внутреннее сопротивление равно \ (\ text {0,1} \) \ (\ text {Ω} \), какова ЭДС батареи и какая мощность рассеивается внутренним сопротивлением батареи?

Примечание

Это вопрос, очень похожий на тот, что вы видели ранее. Это необходимо для того, чтобы выделить Дело в том, что подход к внутреннему сопротивлению строится на том же принципы, с которыми вы уже работали.

Определите, как подойти к проблеме

Нам дана разность потенциалов на ячейке и ток в цепи, а также сопротивления двух из трех резисторов.Мы можем использовать закон Ома для расчета разности потенциалов на известных резисторах. Поскольку резисторы включены в последовательную цепь, разность потенциалов равна \ (V = V_ \ text {1} + V_ \ text {2} + V_ \ text {3} \), и мы можем вычислить \ (V_ \ text {3} \). Теперь мы можем использовать эту информацию, чтобы найти разность потенциалов на неизвестном резисторе \ (R_ \ text {3} \).

Рассчитать разность потенциалов на \ (R_ \ text {1} \)

Используя закон Ома: \ begin {align *} R_ \ text {1} & = \ frac {V_ \ text {1}} {I} \\ I \ cdot R_ \ text {1} & = I \ cdot \ frac {V_ \ text {1}} {I} \\ V_ \ text {1} & = {I} \ cdot {R_ \ text {1}} \\ & = 2 \ cdot 1 \\ V_ \ текст {1} & = \ текст {2} \ текст {V} \ end {align *}

Вычислить разность потенциалов на \ (R_ \ text {2} \)

Снова используя закон Ома: \ begin {align *} R_ \ text {2} & = \ frac {V_ \ text {2}} {I} \\ I \ cdot R_ \ text {2} & = I \ cdot \ frac {V_ \ text {2}} {I} \\ V_ \ text {2} & = {I} \ cdot {R_ \ text {2}} \\ & = 2 \ cdot 3 \\ V_ \ текст {2} & = \ текст {6} \ текст {V} \ end {align *}

Вычислить разность потенциалов на \ (R_ \ text {3} \)

Поскольку разность потенциалов на всех резисторах вместе взятых должна быть такой же, как разность потенциалов на ячейке в последовательной цепи, мы можем найти \ (V_ \ text {3} \), используя: \ begin {align *} V & = V_ \ text {1} + V_ \ text {2} + V_ \ text {3} \\ V_ \ text {3} & = V — V_ \ text {1} — V_ \ text {2} \\ & = 23-2-6 \\ V_ \ текст {3} & = \ текст {15} \ текст {V} \ end {align *}

Найдите сопротивление \ (R_ \ text {3} \)

Нам известна разность потенциалов на \ (R_ \ text {3} \) и ток через нее, поэтому мы можем использовать закон Ома для вычисления значения сопротивления: \ begin {align *} R_ \ text {3} & = \ frac {V_ \ text {3}} {I} \\ & = \ frac {\ text {15}} {\ text {2}} \\ R_ \ text {3} & = \ text {7,5} ~ ​​\ Omega \ end {align *}

Разница потенциалов на внутреннем сопротивлении батареи

Значение ЭДС можно рассчитать по разности потенциалов нагрузки и разности потенциалов на внутреннем сопротивлении.2} {R} \), и мы знаем ток в цепи, внутреннее сопротивление и разность потенциалов на ней, поэтому мы можем использовать любую форму уравнения для мощности:

\ begin {align *} P_r & = V_rI_r ​​\\ & = (\ текст {0,2}) (\ текст {2}) \\ & = \ текст {0,4} \ текст {W} \ end {align *}

Напишите окончательный ответ

  • \ (V_ \ text {1} = \ text {2,0} \ text {V} \)
  • \ (V_ \ text {2} = \ text {6,0} \ text {V} \)
  • \ (V_ \ text {3} = \ text {10,0} \ text {V} \)
  • \ (R_ \ text {3} = \ text {7,5} \ Omega \)
  • \ (\ mathcal {E} = \ text {23,2} \ text {V} \)
  • \ (P_r = \ text {0,4} \ text {W} \)

Рабочий пример 8: Внутреннее сопротивление и резисторы параллельно

Разность потенциалов на батарее составляет 18 В, когда она подключена к двум параллельным резисторам \ (\ text {4,00} \) \ (\ Omega \) и \ (\ text {12,00} \) \ ( \ Omega \) соответственно.Рассчитайте ток через ячейку и через каждый из резисторов. Если внутреннее сопротивление батареи \ (\ text {0,375} \) \ (\ text {Ω} \), какова ЭДС батареи?

Сначала нарисуйте схему перед выполнением любых расчетов

Определите, как подойти к проблеме

Нам нужно определить ток через ячейку и каждый из параллельных резисторов. Нам дана разность потенциалов на ячейке и сопротивления резисторов, поэтому мы можем использовать закон Ома для расчета тока.

Рассчитать ток через ячейку

Чтобы рассчитать ток через элемент, нам сначала нужно определить эквивалентное сопротивление остальной части цепи. Резисторы включены параллельно и поэтому: \ begin {align *} \ frac {\ text {1}} {R} & = \ frac {\ text {1}} {R_ \ text {1}} + \ frac {\ text {1}} {R_ \ text {2}} \ \ & = \ frac {\ text {1}} {\ text {4}} + \ frac {\ text {1}} {\ text {12}} \\ & = \ frac {3 + 1} {\ text {12}} \\ & = \ frac {\ text {4}} {\ text {12}} \\ R & = \ frac {\ text {12}} {\ text {4}} = \ text {3,00} \ \ Omega \ end {выровнять *} Теперь, используя закон Ома, чтобы найти ток через ячейку: \ begin {align *} R & = \ frac {V} {I} \\ I & = \ frac {V} {R} \\ & = \ frac {\ text {18}} {\ text {3}} \\ I & = \ text {6,00} \ text {A} \ end {align *}

Теперь определите ток через один из параллельных резисторов

Мы знаем, что для чисто параллельной конфигурации резисторов разность потенциалов на ячейке такая же, как и разность потенциалов на каждом из параллельных резисторов.Для этой схемы: \ begin {align *} V & = V_ \ text {1} = V_ \ text {2} = \ text {18} \ text {V} \ end {выровнять *} Начнем с вычисления тока через \ (R_ \ text {1} \) по закону Ома: \ begin {align *} R_ \ text {1} & = \ frac {V_ \ text {1}} {I_ \ text {1}} \\ I_ \ text {1} & = \ frac {V_ \ text {1}} {R_ \ text {1}} \\ & = \ frac {\ text {18}} {\ text {4}} \\ I_ \ text {1} & = \ text {4,50} \ text {A} \ end {align *}

Рассчитайте ток через другой параллельный резистор

Мы можем снова использовать закон Ома, чтобы найти ток в \ (R_ \ text {2} \): \ begin {align *} R_ \ text {2} & = \ frac {V_ \ text {2}} {I_ \ text {2}} \\ I_ \ text {2} & = \ frac {V_ \ text {2}} {R_ \ text {2}} \\ & = \ frac {\ text {18}} {\ text {12}} \\ I_ \ text {2} & = \ text {1,50} \ text {A} \ end {выровнять *} Альтернативный метод вычисления \ (I_ \ text {2} \) заключался бы в использовании того факта, что токи через каждый из параллельных резисторов должны составлять общий ток через ячейку: \ begin {align *} I & = I_ \ text {1} + I_ \ text {2} \\ I_ \ text {2} & = I — I_ \ text {1} \\ & = 6 — 4.5 \\ I_ \ text {2} & = \ text {1,5} \ text {A} \ end {align *}

Определить ЭДС

Суммарный ток через батарею — это ток через внутреннее сопротивление батареи. Знание силы тока и сопротивления позволяет нам использовать закон Ома для определения разности потенциалов на внутреннем сопротивлении и, следовательно, ЭДС батареи.

Используя закон Ома, мы можем определить разность потенциалов на внутреннем сопротивлении:

\ begin {align *} V & = I \ cdot r \\ & = \ текст {6} \ cdot \ text {0,375} \\ & = \ текст {2,25} \ текст {V} \ end {выровнять *}

Мы знаем, что ЭДС аккумулятора — это разность потенциалов на выводе, суммированная с разностью потенциалов на внутреннем сопротивлении, поэтому:

\ begin {align *} \ mathcal {E} & = V + Ir \\ & = \ text {18} + \ text {2,25} \\ & = \ текст {20,25} \ текст {V} \ end {align *}

Напишите окончательный ответ

Ток через ячейку равен \ (\ text {6,00} \) \ (\ text {A} \).

Ток через резистор \ (\ text {4,00} \) \ (\ Omega \) равен \ (\ text {4,50} \) \ (\ text {A} \).

Ток через резистор \ (\ text {12,00} \) \ (\ Omega \) равен \ (\ text {1,50} \) \ (\ text {A} \).

ЭДС батареи равна \ (\ text {20,25} \) \ (\ text {V} \).

Рабочий пример 9: Мощность в последовательной и параллельной сетях резисторов

Учитывая следующую схему:

Ток, покидающий батарею, равен \ (\ text {1,07} \) \ (\ text {A} \), общая мощность, рассеиваемая во внешней цепи, равна \ (\ text {6,42} \) \ ( \ text {W} \), отношение полных сопротивлений двух параллельных сетей \ (R_ {P \ text {1}}: R_ {P \ text {2}} \) равно 1: 2, соотношение \ (R_ \ text {1}: R_ \ text {2} \) равно 3: 5 и \ (R_ \ text {3} = \ text {7,00} \ text {Ω} \).

Определите:

  1. разность потенциалов АКБ,
  2. мощность, рассеиваемая в \ (R_ {P \ text {1}} \) и \ (R_ {P \ text {2}} \), и
  3. , если батарея имеет ЭДС, равную \ (\ text {6,50} \) \ (\ text {V} \), каково значение сопротивления каждого резистора и мощность, рассеиваемая в каждом из них.

Что требуется

В этом вопросе вам дается различная информация и предлагается определить мощность, рассеиваемую на каждом резисторе и каждой комбинации резисторов.Обратите внимание, что данная информация в основном относится ко всей цепи. Это подсказка, которую вы должны начать с общей схемы и двигаться вниз к более конкретным элементам схемы.

Расчет разности потенциалов аккумулятора

В первую очередь остановимся на батарее. Нам дана мощность всей цепи, а также ток, покидающий батарею. Мы знаем, что разность потенциалов на клеммах аккумулятора — это разность потенциалов в цепи в целом.

Мы можем использовать соотношение \ (P = VI \) для всей схемы, потому что разность потенциалов такая же, как разность потенциалов на клеммах батареи: \ begin {align *} P & = VI \\ V & = \ frac {P} {I} \\ & = \ frac {\ text {6,42}} {\ text {1,07}} \\ & = \ текст {6,00} \ текст {V} \ end {align *}

Разность потенциалов на батарее равна \ (\ text {6,00} \) \ (\ text {V} \).

Мощность, рассеиваемая в \ (R_ {P \ text {1}} \) и \ (R_ {P \ text {2}} \)

Помните, что мы работаем от общих деталей схемы вниз к деталям для отдельных элементов, это противоположно тому, как вы относились к этой схеме ранее.

Мы можем рассматривать параллельные сети как эквивалентные резисторы, поэтому схема, с которой мы сейчас работаем, выглядит так:

Мы знаем, что ток через два элемента схемы будет одинаковым, потому что это последовательная цепь и что сопротивление всей цепи должно быть: \ (R_ {Ext} = R_ {P \ text {1}} + R_ {P \ text {2}} \). Мы можем определить полное сопротивление по закону Ома для цепи в целом: \ begin {align *} V_ {батарея} & = IR_ {Ext} \\ R_ {Ext} & = \ frac {V_ {аккумулятор}} {I} \\ & = \ frac {\ text {6,00}} {\ text {1,07}} \\ & = \ текст {5,61} \ текст {Ω} \ end {align *}

Мы знаем, что соотношение между \ (R_ {P \ text {1}}: R_ {P \ text {2}} \) равно 1: 2, что означает, что мы знаем: \ begin {align *} R_ {P \ text {1}} & = \ frac {\ text {1}} {\ text {2}} R_ {P \ text {2}} \ \ \ text {и} \\ R_T & = R_ {P \ text {1}} + R_ {P \ text {2}} \\ & = \ frac {\ text {1}} {\ text {2}} R_ {P \ text {2}} + R_ {P \ text {2}} \\ & = \ frac {\ text {3}} {\ text {2}} R_ {P \ text {2}} \\ (\ text {5,61}) & = \ frac {\ text {3}} {\ text {2}} R_ {P \ text {2}} \\ R_ {P \ text {2}} & = \ frac {\ text {2}} {\ text {3}} (\ text {5,61}) \\ R_ {P \ text {2}} & = \ text {3,74} \ text {Ω} \ end {выровнять *} и поэтому: \ begin {align *} R_ {P \ text {1}} & = \ frac {\ text {1}} {\ text {2}} R_ {P \ text {2}} \\ & = \ frac {\ text {1}} {\ text {2}} (3.2 (\ text {3,74}) \\ & = \ текст {4,28} \ текст {W} \ end {выровнять *} Эти значения будут в сумме с исходным значением мощности, которое у нас было для внешней цепи. Если бы они этого не сделали мы бы сделали ошибку в расчетах.

Расчет параллельной сети 1

Теперь мы можем приступить к детальному расчету первого набора параллельных резисторов.

Мы знаем, что соотношение между \ (R _ {\ text {1}}: R _ {\ text {2}} \) составляет 3: 5, что означает, что мы знаем \ (R _ {\ text {1}} = \ frac {\ text {3}} {\ text {5}} R _ {\ text {2}} \).Нам также известно общее сопротивление двух параллельных резисторов в этой сети. это \ (\ text {1,87} \) \ (\ text {Ω} \). Мы можем использовать соотношение между значениями двух резисторов, а также формула для общей сопротивление (\ (\ frac {\ text {1}} {R_PT} = \ frac {\ text {1}} {R_ \ text {1}} + \ frac {\ text {1}} {R_ \ text {2) }} \)) чтобы найти номиналы резисторов: \ begin {align *} \ frac {\ text {1}} {R_ {P \ text {1}}} & = \ frac {\ text {1}} {R_ \ text {1}} + \ frac {\ text {1}} { R_ \ text {2}} \\ \ frac {\ text {1}} {R_ {P \ text {1}}} & = \ frac {\ text {5}} {3R_ \ text {2}} + \ frac {\ text {1}} { R_ \ text {2}} \\ \ frac {\ text {1}} {R_ {P \ text {1}}} & = \ frac {\ text {1}} {R_ \ text {2}} (\ frac {\ text {5}} { \ text {3}} + 1) \\ \ frac {\ text {1}} {R_ {P \ text {1}}} & = \ frac {\ text {1}} {R_ \ text {2}} (\ frac {\ text {5}} { \ text {3}} + \ frac {\ text {3}} {\ text {3}}) \\ \ frac {\ text {1}} {R_ {P \ text {1}}} & = \ frac {\ text {1}} {R_ \ text {2}} \ frac {\ text {8}} {\ текст {3}} \\ R_ \ text {2} & = R_ {P \ text {1}} \ frac {\ text {8}} {\ text {3}} \\ & = (\ text {1,87}) \ frac {\ text {8}} {\ text {3}} \\ & = \ текст {4,99} \ текст {Ω} \ end {выровнять *} Мы также можем вычислить \ (R _ {\ text {1}} \): \ begin {align *} R _ {\ text {1}} & = \ frac {\ text {3}} {\ text {5}} R _ {\ text {2}} \\ & = \ frac {\ text {3}} {\ text {5}} (\ text {4,99}) \\ & = \ текст {2,99} \ текст {Ω} \ end {align *}

Для определения мощности нам нужно рассчитанное нами сопротивление и либо разность потенциалов, либо ток. 2} {\ text {4,99}} \\ & = \ текст {0,80} \ текст {W} \ end {align *}

Параллельная сеть 2 расчета

Теперь мы можем приступить к детальному расчету второго набора параллельных резисторов.

Нам дано \ (R_ \ text {3} = \ text {7,00} \ text {Ω} \), и мы знаем \ (R_ {P \ text {2}} \), поэтому мы можем вычислить \ (R_ \ text {4} \) из: \ begin {align *} \ frac {\ text {1}} {R_ {P \ text {2}}} & = \ frac {\ text {1}} {R_ \ text {3}} + \ frac {\ text {1}} { R_ \ text {4}} \\ \ frac {\ text {1}} {\ text {3,74}} & = \ frac {\ text {1}} {\ text {7,00}} + \ frac {\ text {1}} {R_ \ текст {4}} \\ R_ \ text {4} & = \ text {8,03} \ text {Ω} \ end {align *}

Мы можем вычислить разность потенциалов во второй параллельной сети, вычтя разность потенциалов первой параллельной сети из разности потенциалов батареи, \ (V_ {P \ text {2}} = \ text {6,00} — \ text {2,00} = \ text {4,00} \ text {V} \).2} {\ text {8,03}} \\ & = \ текст {1,99} \ текст {W} \ end {align *}

Внутреннее сопротивление

Мы знаем, что ЭДС батареи равна \ (\ text {6,5} \) \ (\ text {V} \), но что разность потенциалов, измеренная на клеммах, составляет всего \ (\ text {6} \) \ (\ текст {V} \). Разница — это разность потенциалов на внутреннем сопротивлении батареи, и мы можем использовать известный ток и закон Ома для определения внутреннего сопротивления:

\ begin {align *} V & = I \ cdot R \\ R & = \ frac {V} {I} \\ & = \ frac {\ text {0,5}} {\ text {1,07}} \\ & = \ text {0,4672897} \\ & = \ текст {0,47} \ текст {Ω} \ end {выровнять *}

Мощность, рассеиваемая внутренним сопротивлением батареи:

\ begin {align *} P & = VI \\ & = \ текст {0,5} \ cdot \ text {1,07} \\ & = \ текст {0,535} \ текст {W} \ end {align *}

Рабочий пример 10: Внутреннее сопротивление и фары [NSC 2011 Paper 1]

Фара и два ИДЕНТИЧНЫХ задних фонаря скутера подключены параллельно к батарее с неизвестным внутренним сопротивлением, как показано на упрощенной принципиальной схеме ниже.Фара имеет сопротивление \ (\ text {2,4} \) \ (\ text {Ω} \) и управляется переключателем \ (\ textbf {S} _1 \). Задние фонари управляются переключателем \ (\ textbf {S} _2 \). Сопротивлением соединительных проводов можно пренебречь.

На приведенном рядом графике показана разность потенциалов на клеммах батареи до и после включения переключателя \ (\ textbf {S} _1 \) (пока переключатель \ (\ textbf {S} _2 \) открыт). Переключатель \ (\ textbf {S} _1 \) закрывается в момент \ (\ textbf {t} _1 \).

  1. Используйте график, чтобы определить ЭДС аккумулятора.

    (1 балл)

  2. ПРИ ТОЛЬКО ПЕРЕКЛЮЧАТЕЛЬ \ (\ textbf {S} _1 \) ЗАКРЫТО, рассчитайте следующее:

    1. Ток через фару

      (3 балла)

    2. Внутреннее сопротивление \ (r \) батареи

      (3 балла)

  3. ОБЕ ПЕРЕКЛЮЧАТЕЛИ \ (\ textbf {S} _1 \) И \ (\ textbf {S} _2 \) ТЕПЕРЬ ЗАКРЫТЫ.В течение этого периода аккумулятор обеспечивает ток \ (\ text {6} \) \ (\ text {A} \).

    Рассчитайте сопротивление каждого заднего фонаря.

    (5 баллов)

  4. Как повлияет на показания вольтметра, если фара перегорит? (Оба переключателя \ (\ textbf {S} _1 \) и \ (\ textbf {S} _2 \) все еще закрыты.)

    Запишите только УВЕЛИЧИВАЕТ, УМЕНЬШАЕТСЯ или ОСТАЕТСЯ ОДИН ТО ЖЕ.

    Дайте объяснение.

    (3 балла)

Вопрос 1

\ (\ text {12} \) \ (\ text {V} \)

(1 балл)

Вопрос 2.1

Вариант 1:

\ begin {align *} I & = \ frac {V} {R} \\ & = \ frac {\ text {9,6}} {\ text {2,4}} \\ & = \ текст {4 A} \ end {выровнять *}

Вариант 2:

\ begin {align *} \ text {emf} & = IR + Ir \\ 12 & = I (\ text {2,4}) + \ text {2,4} \\ \ поэтому I & = \ text {4 A} \ end {выровнять *}

(3 балла)

Вопрос 2.2

Вариант 1:

\ begin {align *} \ text {emf} & = IR + Ir \\ 12 & = \ text {9,4} + 4r \\ r & = \ текст {0,6} \ \ Omega \ end {выровнять *}

Вариант 2:

\ begin {align *} V_ {потеряно} & = Ir \\ \ text {2,4} & = \ text {4} r \\ \ поэтому r & = \ text {0,6} \ \ Omega \ end {выровнять *}

Вариант 3:

\ begin {align *} \ text {emf} & = I (R + r) \\ \ text {12} & = \ text {4} (\ text {2,4} + r) \\ \ поэтому r & = \ text {0,6} \ \ Omega \ end {выровнять *}

(3 балла)

Вопрос 3

Вариант 1:

\ begin {align *} \ text {emf} & = IR + Ir \\ \ text {12} & = \ text {6} (R + \ text {0,6}) \\ R _ {\ text {ext}} & = \ text {1,4} \ \ Omega \ конец {выравнивание *} \ begin {выравнивание *} \ frac {1} {R} & = \ frac {1} {R_ {1}} + \ frac {1} {R_ {2}} \\ \ frac {1} {\ text {1,4}} & = \ frac {1} {\ text {2,4}} + \ frac {1} {R} \\ R & = \ текст {3,36} \ \ Omega \ end {выровнять *}

Каждый задний фонарь: \ (R = \ text {1,68} \ \ Omega \)

Вариант 2:

\ begin {align *} \ text {Emf} & = V _ {\ text {terminal}} + Ir \\ 12 & = V _ {\ text {терминал}} + 6 (\ text {0,6}) \\ \ поэтому V _ {\ text {terminal}} & = \ text {8,4} \ text {V} \ конец {выравнивание *} \ begin {выравнивание *} I _ {\ text {2,4} \ \ Omega} & = \ frac {V} {R} \\ & = \ frac {\ text {8,4}} {\ text {2,4}} \\ & = \ text {3,5 A} \ конец {выравнивание *} \ begin {выравнивание *} I _ {\ text {задние фонари}} & = 6 — \ text {3,5} \\ & = \ текст {2,5} \ текст {A} \\ R _ {\ text {задние фонари}} & = \ frac {V} {I} \\ & = \ frac {\ text {8,4}} {\ text {2,5}} \\ & = \ текст {3,36} \ \ Omega \\ R _ {\ text {задний фонарь}} & = \ text {1,68} \ \ Omega \ end {выровнять *}

Вариант 3:

\ begin {align *} V & = IR \\ \ text {12} & = \ text {6} (R) \\ R _ {\ text {ext}} & = 2 \ \ Omega \ конец {выравнивание *} \ begin {выравнивание *} R _ {\ text {parallel}} & = 2 — \ text {0,6} \\ & = \ текст {1,4} \ \ Omega \\ \ frac {1} {R} & = \ frac {1} {R_ {1}} + \ frac {1} {R_ {2}} \\ \ frac {1} {\ text {1,4}} & = \ frac {1} {\ text {2,4}} + \ frac {1} {R} \\ R & = \ текст {3,36} \ \ Omega \ end {выровнять *}

Каждый задний фонарь: \ (R = \ text {1,68} \ \ Omega \)

Вариант 4:

Для параллельной комбинации: \ (I_ {1} + I_ {2} = 6 \ text {A} \)

\ begin {align *} \ поэтому \ frac {V} {\ text {2,4}} + \ frac {V} {R _ {\ text {задние фонари}}} & = \ text {6} \\ \ text {8,4} \ left (\ frac {1} {\ text {2,4}} + \ frac {1} {R _ {\ text {задние фонари}}} \ right) & = \ text {6 } \\ \ поэтому R _ {\ text {задние фонари}} & = \ text {3,36} \ \ Omega \\ R _ {\ text {задний фонарь}} & = \ text {1,68} \ \ Omega \ end {выровнять *}

(5 баллов)

Вопрос 4

Увеличивается

Сопротивление увеличивается, а ток уменьшается.Поэтому \ (Ir \) (потерянное вольт) должно уменьшаться, что приводит к увеличению напряжения.

(3 балла)

[ИТОГО: 15 баллов]

ЭДС и внутреннее сопротивление

ЭДС и внутреннее сопротивление
следующий: резисторы в серии и вверх: электрический ток Предыдущий: Сопротивление и удельное сопротивление Теперь настоящие батареи изготавливаются из материалов с ненулевым удельным сопротивлением. Отсюда следует, что настоящие батареи — это не просто источники чистого напряжения.Они также обладают внутренние сопротивления . Между прочим, чистое напряжение Источник обычно обозначается как ЭДС (что означает электродвижущую силу ). Конечно, ЭДС измеряется в вольтах. Аккумулятор можно смоделировать как ЭДС, включенную последовательно с резистором. , который представляет собой его внутреннее сопротивление. Предположим, что такие батарея используется для управления током через внешний нагрузочный резистор, так как показано на рис.17. Обратите внимание, что на принципиальных схемах ЭДС представлена ​​в виде двух близко расположенных параллельных линии неравной длины.Электрический потенциал более длинной линии больше, чем тот из более коротких по вольтам. Резистор представлен как зигзагообразная линия.
Рисунок 17: Батарея ЭДС и внутреннего сопротивления подключена к нагрузочному резистору сопротивления.

Рассмотрим аккумулятор на рисунке. Напряжение аккумулятора равно определяется как разница в электрическом потенциале между его положительным и отрицательные клеммы: i.е. , точки и, соответственно. Когда мы переходим от к , электрический потенциал увеличивается на вольт, когда мы пересекаем ЭДС, но затем уменьшается на вольт, когда мы пересекаем внутренний резистор. Падение напряжения на резисторе следует из закона Ома, из которого следует, что падение напряжения на резисторе, несущем ток , находится в том направлении, в котором текущие потоки. Таким образом, напряжение аккумулятора связано с его ЭДС. и внутреннее сопротивление через

(133)

Обычно мы думаем, что ЭДС батареи по существу постоянная (поскольку она зависит только от химической реакции, происходящей внутри батареи, которая преобразует химическая энергия в электрическую), поэтому мы должны заключить, что напряжение батарея на самом деле уменьшается по мере увеличения тока, потребляемого от нее.Фактически, напряжение равно только ЭДС при пренебрежимо малом токе. Текущий розыгрыш от аккумулятора обычно не может превышать критическое значение
(134)

поскольку напряжение становится отрицательным (что может произойти только если резистор нагрузки также отрицательный: это практически невозможно). Отсюда следует, что если мы закоротим аккумулятор, подключив его положительные и отрицательные клеммы вместе с использованием проводящего провода с незначительным сопротивлением, ток, потребляемый батареей, ограничен ее внутренним сопротивлением.Фактически в этом случае сила тока равна максимально возможной. Текущий .

Настоящая батарея обычно характеризуется его ЭДС (, т.е. , его напряжение при нулевом токе) и максимальный ток, который он может подавать. Например, стандартный сухой элемент (, т.е. , своего рода аккумулятор, используемый для питания калькуляторов и фонарей) обычно рассчитан на и скажи) . Таким образом, ничего действительно катастрофического не произойдет. произойдет, если мы закоротим сухой элемент.Мы разрядим батарею через сравнительно короткий промежуток времени, но опасно большой ток не будет поток. С другой стороны, автомобильный аккумулятор обычно рассчитывается на и что-то вроде (такой ток нужен для запустить стартер). Понятно, что автомобильный аккумулятор должен иметь много более низкое внутреннее сопротивление, чем у сухого элемента. Отсюда следует, что если мы были достаточно глупы, чтобы замкнуть автомобильный аккумулятор, в результате довольно катастрофически (представьте себе всю энергию, необходимую для запуска двигателя автомобиль собирается тонким проводом, соединяющим клеммы аккумулятора вместе).



следующий: резисторы в серии и вверх: электрический ток Предыдущий: Сопротивление и удельное сопротивление
Ричард Фицпатрик 2007-07-14

ЭДС, подключение резисторов последовательно и параллельно — интерактивная практика

Поток электрического тока

Преобразование энергии на двух концах …

В электрической цепи всегда присутствует источник энергии и нагрузка . Первый генерирует энергию, а второй расходует ее. Источником может быть элемент, батарея, динамо-машина или даже солнечная батарея. Нагрузкой может быть резистор, лампочка, вентилятор или нагреватель. Мы знаем, что пока происходят преобразования энергии в источнике и в нагрузке, электрический ток течет по цепи от точки с более высоким потенциалом к ​​точке с более низким потенциалом.

В приведенной выше анимации катящиеся шары имитируют электрический ток, опускаясь от точки с высокой потенциальной энергией к точке с низкой потенциальной энергией.Точно так же и ток, переходит от положительной клеммы — при более высоком потенциале — к отрицательной — при более низком потенциале. Однако, когда шары достигают ступней человека, они должны перейти от точки с более низкой потенциальной энергией к точке с более высокой потенциальной энергией. Это невозможно без вмешательство мужчины. Поэтому он наклоняется и поднимает их, чтобы они продолжали двигаться, превращая свою химическую энергию в потенциальную.

Когда заряды, переносящие электрический ток, достигают отрицательной клеммы ячейки, они сталкиваются с одним и тем же вызовом.Таким образом, химическая энергия в батарее превращается в электрическую, чтобы обеспечить заряды энергией для преодоления препятствий. Вот почему человек устает, а батарея разряжается через некоторое время, поскольку их соответствующие энергии превратились в разные формы.

Электродвижущая сила — ЭДС

Количество химической энергии, которое превратилось в электрическую энергию для перемещения + 1C по цепи, называется Электродвижущая сила клетки.

Единицы: Вольт

Мощность устройства

Энергия, потребляемая устройством за единицу времени, называется его мощностью.
Единицы:
P = E / t = J / t = Watts
Если напряжение равно V и заряд проходит через него Q за время t,
E = QV
Итак, P = QV / t = ItV / t = VI
P = VI
P = Вт; V = Вольт; Я = А

Мощность устройства должна быть указана вместе с напряжением, при котором она действительна.
60W, 240V означает, что мощность 60W вырабатывается при 240V.

Например, 1

Номиналы лампы 60 Вт, 240 В. Найдите его сопротивление.
P = VI
60 = 240I
I = 1/4 A
V = IR
240 = (1/4) R
R = 960 Ом.

Например, 2

Номиналы утюга — 1200Вт, 240В. Найдите ток и энергию, израсходованную за час.
P = VI
1200 = 240I
I = 5A.
E = Pt
E = 1200 X 3600
E = 4,32×10 6 J.

Например, 3

Номиналы лампы 60 Вт, 240 В. Из-за отключения электроэнергии напряжение падает до 200 В. Найдите новую мощность лампочки. Что бы вы заметили в лампочке?
P = VI
60 = 240I
I = 1/4 A
V = IR
240 = (1/4) R
R = 960Ω
При работе под новым напряжением
V = IR
200 = I x 960
I = 5/24 A
P = VI
P = 200 X 5/24
P = 41.7W
Лампа станет тусклее.

Доказательства наличия внутреннего сопротивления ячейки

Предположим, что заряд Q проходит по цепи за время t. ЭДС ячейки, внешнее сопротивление и ток равны E, R и I.
Энергия, производимая элементом = QE
Энергия, потребляемая внешним резистором = QV t
На практике было отмечено QV t Следовательно, мы должны учитывать потерю энергии, кроме что произошло на внешнем резисторе.Это, безусловно, вызвано сопротивление присутствует в клетке. Он называется внутренним сопротивлением (r).
Таким образом, новое уравнение энергии принимает следующий вид:
QE = QV внутренний + QV внешний
Поскольку Q = It и V = IR => ItE = Irt + IRt
E = Ir + IR
E = I (r + R)
IR = E — Ir
Это разность потенциалов на внешнем резисторе (нагрузке). Поскольку вольтметр, подключенный к ячейке, показывает это, вместо ЭДС, он известен как Разница конечных потенциалов — В т .
Итак, В t = E — Ir
В разомкнутой цепи I = 0. Следовательно,
В t = E.

Например,

ЭДС ячейки составляет 12 В, а ее внутреннее сопротивление 2 Ом. Найдите ток и разность потенциалов на клеммах ячейки, если она подключена к внешнему резистору 4 Ом.
E = I (R + r)
12 = I (4 + 2)
I = 2A
V t = E — Ir
V t = 12 — 2×2 = 8
V t = 8V.

Соединение ячеек

Элементы могут быть подключены последовательно, или параллельно или их комбинация.

Соединение в серии

Ячейки соединены таким образом, чтобы ток через каждую из них был одинаковым.

Параллельное подключение

Ячейки ЭДС соединены таким образом, что через каждую проходит равный ток.

Подключение резисторов

Резисторы
серии

Если резисторы подключены таким образом, что ток через каждый из них одинаков, они считаются включенными последовательно.

Единственный резистор, который может заменить комбинацию, должен создавать напряжение V, когда ток через него равен I.
Для отдельных резисторов V = V 1 + V 2
V = IR 1 + IR 2
V = I (R 1 + R 2 )
Для эквивалентного резистора — заменитель,
V = IR T
IR T = I (R 1 + R 2 )
R T = (R 1 + R 2 )

Параллельные резисторы

Если резисторы подключены таким образом, что напряжение на них одинаково, они считаются параллельными.

Единственный резистор, который может заменить комбинацию, должен создавать напряжение V, когда ток через него равен I.
Для отдельных резисторов I = a + b
I = V / R 1 + V / R 2
V = V (1 / R 1 + 1 / R 2 )
Для эквивалентного резистора — заменитель,
I = V / R T
V / R T = V / (1 / R 1 + 1 / R 2 )
1 / R T = 1 / R 1 + 1 / R 2

E.g.1

Найдите полное сопротивление следующей цепи.

Общее сопротивление по xy:
1 / R T = (1/6) + (1/3)
1 / R T = (1 + 2) / 6 = 2/6 = 1/2
R T = 2 Ом
Полное сопротивление ветви = 2 + 4 = 6 Ом
Полное сопротивление трех ветвей, которые теперь параллельны
1 / R T = (1/6) + (1/15) + ( 1/10)
1 / R T = (5 + 2 + 3) / 30 = 10/30 = 1/3
R T = 3 Ом

E.g.2

Когда два резистора соединены последовательно, общее сопротивление составляет 25 Ом. Если они соединены параллельно, общее сопротивление составляет 6 Ом. Найдите сопротивление каждого.
Когда они включены последовательно,
R T = x + y = 25 Ом
Когда они подключены параллельно,
1 / R T = (1 / x) + (1 / y)
1 / R T = (x + y) / xy = 1/6
6 (x + y) = xy
6 X 25 = xy => xy = 150 => x = 150 / x
Итак, 150 / x + x = 25
x 2 + 150 = 25x
x 2 — 25x + 150 = 0
(x — 15) (x — 10) = 0
x = 15 или x = 10
Сопротивление каждого резистора составляет 15 Ом или 10 Ом

E.g.3

Найдите полное сопротивление следующей цепи и ток.

Поскольку все резисторы включены последовательно, общее сопротивление будет следующим:
R T = 1 + 2 + 3 + 4 = 10 Ом
Общий ток = 20/10 = 2A
Этот ток протекает через каждый резистор, как они есть последовательно.

Например, 4

Найдите полное сопротивление следующей цепи и токи в каждой ветви.

Суммарное сопротивление по AB:
1 / R T = (1/6) + (1/3)
1 / R T = (1 + 2) / 6 = 2/6 = 1/2
R T = 2 Ом
Общее сопротивление цепи = 2 + 2 + 1 = 5 Ом
Общий ток = 15/5 = 3A
Этот ток делится на A обратно пропорционально сопротивлению каждой ветви — чем больше сопротивление , тем меньше ток, проходящий через него.
Поскольку два сопротивления параллельны,
6a = 3b
b = 2a
a + b = 3
3a = 3 => a = 1A; б = 2А.
Остальные резисторы получают ток 3А.

Дополнительные вопросы по резисторам в схемах

Найдите полное сопротивление следующих цепей:

Ответ: 5 Ом

Ответ: 6 Ом

В следующей схеме резисторы красного цвета.

alexxlab

Добавить комментарий

Ваш адрес email не будет опубликован. Обязательные поля помечены *